10 Pulmunology

Download as pdf or txt
Download as pdf or txt
You are on page 1of 111

A COMPLETE HANDBOOK FOR

KROK - II
CARDIOLOGY

| DR.SHEKHAR MAAN SHRESTHA |


CONTENTS
Table of Contents
SARCOIDOSIS..............................................................................................................................5
MCQS...................................................................................................................................... 5
BRONCHIOLITIS.......................................................................................................................... 5
MCQS...................................................................................................................................... 5
Stenosing laryngotracheobronchitis......................................................................................... 6
MCQS...................................................................................................................................... 7
bronchitis................................................................................................................................... 10
MCQS.................................................................................................................................... 10
pneumonia..................................................................................................................................12
MCQS.................................................................................................................................... 12
tuberculosis............................................................................................................................... 19
MCQS.................................................................................................................................... 19
Kartagener syndrome................................................................................................................23
MCQS.................................................................................................................................... 24
Mucoviscidosis (cystic fibrosis).............................................................................................. 25
MCQS.................................................................................................................................... 25
Bronchiectasis........................................................................................................................... 27
MCQS.................................................................................................................................... 27
lung cancer.................................................................................................................................31
MCQS.................................................................................................................................... 32
LUNG GANGRENE.....................................................................................................................35
MCQS.................................................................................................................................... 35
ASTHMA..................................................................................................................................... 36
MCQS.................................................................................................................................... 36
PNEUMOCONIOSIS................................................................................................................... 42
MCQS.................................................................................................................................... 42
Chronic obstructive lung disease............................................................................................ 45
MCQS.................................................................................................................................... 45
Allergic rhinitis...........................................................................................................................46
MCQS.................................................................................................................................... 46
Pulmonary edema......................................................................................................................47
MCQS.................................................................................................................................... 47
Airway foreign body.................................................................................................................. 48
MCQS.................................................................................................................................... 48
Somatoform autonomic dysfunction....................................................................................... 51
MCQS.................................................................................................................................... 51
PULMONARY EMBOLISM......................................................................................................... 51
MCQS.................................................................................................................................... 52
DIAGNOSTICS IN PNEUMOLOGY............................................................................................ 53
MCQS.................................................................................................................................... 53
plague......................................................................................................................................... 59
MCQS.................................................................................................................................... 59
ASPHYXIA.................................................................................................................................. 61
MCQS.................................................................................................................................... 61
TRACHEOESOPHAGEAL FISTULA..........................................................................................62
MCQS.................................................................................................................................... 62
PLEURISY...................................................................................................................................63
MCQS.................................................................................................................................... 63
PNEUMOTHORAX......................................................................................................................64
MCQS.................................................................................................................................... 64
PLEURAL EMPYEMA.................................................................................................................81
MCQS.................................................................................................................................... 81
MEDIASTINITIS.......................................................................................................................... 82
MCQS.................................................................................................................................... 82
PLEURITIS.................................................................................................................................. 84
MCQS.................................................................................................................................... 84
PULMONARY INFARCTION.......................................................................................................85
MCQS.................................................................................................................................... 85
PULMONARY ABSCESS........................................................................................................... 86
MCQS.................................................................................................................................... 86
RESPIRATORY DISTRESS........................................................................................................ 89
MCQS.................................................................................................................................... 89
ATELECTASIS............................................................................................................................ 90
MCQS.................................................................................................................................... 90
LARYNGEAL EDEMA................................................................................................................ 91
MCQS.................................................................................................................................... 91
PLEURAL EFFUSION.................................................................................................................92
MCQS.................................................................................................................................... 92
PULMONARY FIBROSIS............................................................................................................93
MCQS.................................................................................................................................... 93
PULMONARY EDEMA................................................................................................................94
MCQS.................................................................................................................................... 94
REFLEX.......................................................................................................................................95
MCQS.................................................................................................................................... 95
RESPIRATORY FAILURE...........................................................................................................96
MCQS.................................................................................................................................... 96
Physiology of pregnancy.......................................................................................................... 96
MCQS.................................................................................................................................... 96
Physiology of pregnancy.......................................................................................................... 97
MCQS.................................................................................................................................... 97
DERMATITIS............................................................................................................................... 97
MCQS.................................................................................................................................... 97
Physiology of pregnancy.......................................................................................................... 98
MCQS.................................................................................................................................... 98
OPHTHALMOLOGY................................................................................................................... 98
MCQS.................................................................................................................................... 98
PNEUMOCISTA.......................................................................................................................... 98
MCQS.................................................................................................................................... 99
CANCER OF LUNG.................................................................................................................... 99
MCQS.................................................................................................................................... 99
INFLUENZA.............................................................................................................................. 100
MCQS.................................................................................................................................. 100
PERTUSIS.................................................................................................................................102
MCQS.................................................................................................................................. 102
RESPIRATORY FAILURE.........................................................................................................103
MCQS.................................................................................................................................. 103
ALVEOLITIS..............................................................................................................................104
MCQS.................................................................................................................................. 104
Physiology of pregnancy........................................................................................................ 105
MCQS.................................................................................................................................. 105
Physiology of pregnancy........................................................................................................ 105
MCQS.................................................................................................................................. 105
Physiology of pregnancy........................................................................................................ 106
MCQS.................................................................................................................................. 106
Physiology of pregnancy........................................................................................................ 106
MCQS.................................................................................................................................. 106
Physiology of pregnancy........................................................................................................ 106
MCQS.................................................................................................................................. 106
Physiology of pregnancy........................................................................................................ 107
MCQS.................................................................................................................................. 107
SHOCK......................................................................................................................................107
MCQS.................................................................................................................................. 107
HEART FAILURE...................................................................................................................... 108
MCQS.................................................................................................................................. 108
CARDIOGENIC SHOCK........................................................................................................... 109
MCQS.................................................................................................................................. 109
Complicated hypertensic crisis..............................................................................................109
MCQS.................................................................................................................................. 110
SARCOIDOSIS
To replace the placeholder text on this page, you can just select it all and then start typing. But
don’t do that just yet!

MCQS

During routine medical examination a 35-year-old woman presents with enlarged cervical and
mediastinal lymph nodes. Her overall health is satisfactory. ESR is 30 mm/hour. Cervical node
biopsy was performed. In the specimen there are granulomas composed of epithelial and giant
cells, no caseous necrosis detected. What is the most likely diagnosis?

● Sarcoidosis
● Lymphogranulomatosis
● Infectious mononucleosis
● Nonspecific lymphadenitis
● Lymph node tuberculosis

BRONCHIOLITIS
To replace the placeholder text on this page, you can just select it all and then start typing. But
don’t do that just yet!

MCQS

A 3-month-old girl has rhinitis, dyspnea, dry cough. She has been sick for 2 days. Objectively: pale
skin, acrocyanosis, hypopnoe; breathing rate is 80/min.; over the whole pulmonary surface there is
vesiculotympanitic (bandbox) resonance observed with numerous bubbling crackles. The most
likely diagnosis is:

100% Acute bronchiolitis

0% Pneumonia

0% Mucoviscidosis

0% Foreign body in airways


0% Acute bronchitis

A child is 4 months old. The disease onset was acute and manifested as a temperature of 37.8°C
and a slight cough. On the third day after the onset, the cough intensified and became
accompanied by dyspnea. Percussion detects a tympanic sound over the lungs, while auscultation
reveals numerous wet fine bubbling and sibilant wheezes during the exhalation. Make the
provisional diagnosis:

Focal pneumonia

Bronchopneumonia

Acute bronchiolitis*

Acute bronchitis

Obstructive bronchitis

A 3 m.o. child fell seriously ill, body temperature rised up to 37,80C, there is semicough. On the
3-rd day the cough grew worse, dyspnea appeared. On percussion: tympanic sound above lungs,
on auscultation: a lot of fine moist and wheezing rales during expiration. What is the most
probable diagnosis?

100% Acute respiratory viral infection, bronchiolitis

0% Acute respiratory viral infection, bronchopneumonia

0% Acute respiratory viral infection, bronchitis

0% Acute respiratory viral infection, bronchitis with asthmatic component

0% Acute respiratory viral infection, focal pneumonia

Stenosing laryngotracheobronchitis
To replace the placeholder text on this page, you can just select it all and then start typing. But
don’t do that just yet!
MCQS

A 6-year-old boy was brought to the emergency room with a 3-hour history of fever up to 39,5oC
and sore throat. The child looks alert, anxious and has a mild inspiratory stridor. You should
immediately:

100% Prepare to establish an airway

0% Obtain an arterial blood gas and start an IV line

0% Order a chest x-ray and lateral view of the neck

0% Examine the throat and obtain a culture

0% Admit the child and place him in a mist tent

A 3 y.o. girl has had a temperature rise up to 380C, rhinitis, dry superficial cough, flabbiness,
appetite loss. Palpation didn’t reveal any changes over her lungs. Percussion sound has a wooden
resonance, auscultation revealed puerile breathing, no rales. In blood: leukopenia, lymphocytosis,
increased ESR. What is the most probable diagnosis?

100% Acute simple tracheitis

0% Acute obstructive bronchitis

0% Recurrent bronchitis, acute condition

0% Acute simple bronchitis

0% Bilateral microfocal pneumonia

A 47-year-old man developed the signs of decompensated laryngeal stenosis

against the background of acute phlegmonous laryngitis. He presents with inspiratory

dyspnea at rest, forced position, cyanotic skin covered in cold sweat, tachycardia,

deficient pulse, and low blood pressure. What urgent treatment tactics should be

chosen?
Administration of glucocorticoid hormones

Tracheostomy*

Oxygen therapy

Intravenous administration of dehydrating agents

Oral administration of hyposensitization substances and broncholytics

A child is 2 years old. The child complains of hoarse voice, dyspnea with obstructed inspiration.
The disease started 3 days ago from dry cough and nose stuffiness. Objectively: general condition
is unbalanced, stridor is present. The child’s skin is pale. Body temperature is 37, 7oC. The palatine
arches are hyperemic. There is no deposit. Heart sounds are rhythmic. Auscultation of lungs
reveals rough breathing sounds, crepitation is absent. Parainfluenza virus has been detected in
nasopharynx lavage. What is the most likely diagnosis?

100% Acute laryngotracheitis

0% Epiglottitis

0% Foreign body

0% Diphtheria

0% Laryngospasm

A 1-year-old child with a case of URTI suddenly developed noisy respirations with difficult
inspiration, intercostal retractions, and barking cough on the 2nd night after the disease onset.
What is the most likely diagnosis?

Acute bronchiolitis

Acute pulmonary inflammation

Bronchial asthma

Acute bronchitis

Stenosing laryngotracheobronchitis*
At night a 2-year-old child with upper respiratory tract infection suddenly

developed dyspnea with labored inspiration. Objectively the skin is pale, perioral

cyanosis and slight acrocyanosis are observed. Breathing is loud, respiration rate is

32/min. Jugular, supra- and infraclavicular fossae retract during breathing Respiration

is coarse on auscultation. Heart sounds are clear and sonorous, heart rate is 120/min.

What condition was complicated by the development of the upper respiratory tract

infection?

Airway foreign body

Bronchiolitis

Obstructive bronchitis

Bronchial asthma

Stenosing laryngotracheitis*

An infant aged 1 year on the third day of common cold at night developed inspiratory stridor,
hoarse voice and barking cough. Physical examination revealed suprasternal and intercostal chest
retractions. There is a bluish skin discoloration moistly seen over the upper lip. The respiratory
rate is 52 per min and pulse- 122 bpm. The body temperature is 37,50C. What disease does the
infant have?

100% Acute infectious croup due to viral laryngotracheitis

0% Acute laryngitis

0% Bronchopneumonia without complications

0% Acute bronchiolitis with respiratory distress

0% Acute epiglottitis
A 57-year-old patient complains of sensation of dryness and pain during swallowing, frequent
unbearable cough, the voice is hoarse. Disease onset was abrupt. On laryngoscopy: laryngeal
mucosa is hyperemic, vocal folds are swollen, laryngeal lumen contains viscous secretion. What
diagnosis is it?

100% Acute laryngitis

0% Acute stenosing laryngotracheitis

0% Bronchial asthma

0% Flegmonous laryngitis

0% Laryngeal diphtheria

A 9-year-old patient has measles. On the 6th day after the rash appeared, the boy developed a
condition manifested by dyspnea, barking cough, stenotic respiration. Objectively: the rash on the
face, neck and torso turned brown. There is a branny desquamation. Respiratory rate is 22/min.
What complication should be diagnosed?

100% Laryngotracheitis

0% Bronchitis

0% Pneumonia

0% Pharyngitis

0% Quinsy

bronchitis
To replace the placeholder text on this page, you can just select it all and then start typing. But
don’t do that just yet!

MCQS

A 65-year-old male patient complains of dyspnea that is getting worse with exertion, morning
cough with expectoration of mucous sputum. For about 15 years, he has been subject to regular
medical check-up for chronic bronchitis. The patient takes berodual (16 inhaled doses per day).
Objectively: body temperature is 36,8°C, RR- 24/min, Ps- 110/min, AP- 145/90 mm Hg.
Auscultation reveals a lot of dry rales above the lungs. FEV1- 65%. What is the optimal tactics of
further management of the patient?

100% To administer inhalation corticosteroids

0% To administer antibiotics

0% To administer theophylline

0% To increase the daily dose of berodual

0% To include short-acting /32-agonists in the therapy

A 3-year-old child has a cough and runny nose. Two other family members have the same signs.
On the third day after the onset of the disease, the cough intensified and became dry and
persistent. The temperature increased to 37.8oC. The act of breathing involves the auxiliary
muscles. Percussion produces a banbox resonance bilaterally in the lungs. Breathing is harsh,
expiration is prolonged, there are moderate and large bubbling wheezes. The wheezes are diffuse
and mostly wet, though in some places they are dry. Make the right diagnosis:

Acute bronchiolitis

Bilateral bronchopneumonia

Stenosing laryngotracheitis

Bronchial asthma

Acute obstructive bronchitis*

For the last 3 days a 2-year-old girl has been presenting with fever of 38.0oC, runny nose, dry
cough, inertness, and appetite loss. Percussion sound has a bandbox resonance. Auscultation
detects puerile breathing without wheezing. In the blood there are leukopenia, lymphocytosis,
and accelerated ESR. Make the right diagnosis:

Acute obstructive bronchitis

Acute tracheitis*
Acute bronchitis

Recurrent bronchitis, exacerbation phase

Bilateral microfocal pneumonia

A patient has been suffering from morning cough accompanied by discharge of small amount of
sputum, dyspnea for 8 years. He has been smoking for 10 years. Objectively: cyanosis, prolonged
expiration, dry rales. What is the most probable diagnosis?

100% Chronic obstructive bronchitis

0% Chronic non-obstructive bronchitis

0% Idiopatic fibrosing alveolitis

0% Multiple bronchiectasis

0% Bronchial asthma

pneumonia
To replace the placeholder text on this page, you can just select it all and then start typing. But
don’t do that just yet!

MCQS

A 5-year-old boy has a history of repeated pneumonia, frequent acute respiratory viral diseases.
Objectively: exertional dyspnea, minor fatigabilty. There is a systolic murmur having its epicenter
in the IV intercostal space on the left. Left relative dullness is found along the midclavicular line.
According to the findings of instrumental methods of examination (electrocardiography,
echocardiography), the patient has been diagnosed with ventricular septal defect,
subcompensation stage. What is the main method of treatment?

100% Operative therapy

0% Phytotherapy
0% Does not require treatment

0% Conservative treatment

0% Indomethacin

A 72-year-old man with pneumonia complains of marked dyspnea, chest pain,

severe cough with expectoration, is 39.5-40°C, no urination for a whole day.

Objectively the patient is conscious. Respiratory rate is 36/min. Over the right lower

pulmonary lobe percussion sound is dull: on auscultation there is bronchial respiration

and numerous moist crackles. Blood pressure is 80/60 mm Hg. Heart rate is 120/min.

Heart sounds are muffled, there is tachycardia. What tactics should the family doctor

choose in the management of this patient?

Hospitalization into the intensive care unit*

Treatment in the day patient facility

Hospitalization into the pulmonology unit

Hospitalization into the neurology unit

Outpatient treatment

A 22-year-old patient is a clerk. His working day runs in a conditioned room. In summer he was
taken by an acute disease with the following symptoms: fever, dyspnea, dry cough, pleural pain,
myalgia, arthralgia. Objectively: moist rales on the right, pleural friction rub. X-ray picture showed
infiltration of the inferior lobe. In blood: WBC - 11 • 109/l, stab neutrophils - 6%, segmented
neutrophils - 70%, lymphocytes - 8%, ESR - 42 mm/h. What is the ethiological factor of
pneumonia?

100% Legionella

0% Mycoplasm
0% Streptococcus

0% Staphylococcus

0% Pneumococcus

A 40-year-old patient presents with a cough in the morning with the production of mucopurulent
sputum and elevated body temperature up to 376°C He has been smoking since he was 17 years
old. Objectively, auscultation detects coarse respirations and diffuse dry crackles in the lungs.
Complete blood count: leukocytes – 12 • 109/L, ESR – 19 mm/hour. Bronchoscopy shows purulent
catarrhal endobronchitis. Make the diagnosis:

Bronchial asthma

Pulmonary tuberculosis

Community-acquired pneumonia*

Lung cancer

Chronic bronchitis

A 40-year-old patient complains of fever up to 39oC, cough with sputum and blood admixtures,
dyspnea, weakness, herpetic rash on the lips. Objectively: respiration rate - 32/min. Under the
shoulder blade on the right the increased vocal fremitus and dullness of percussion sound were
revealed. Auscultation revealed bronchial respiration. Blood count: WBCs - 14 • 109/l, ESR - 35
mm/h. What is the provisional diagnosis?

100% Right-sided croupous pneumonia

0% Focal right-sided pneumonia

0% Cavernous tuberculosis of the right lung

0% Lung cancer

0% Exudative pleuritis
A 56 year old woman has an acute onset of fever up to 39°C with chills, cough, and pain on
respiration in the right side of her chest. On physical examination: HR - 90/min, BP- 95/60 mm
Hg, RR-26/min. There is dullness over the right lung on percussion. On X-ray: infiltrate in the
right middle lobe of the lung. What is the diagnosis?

100% Community-acquired lobar pneumonia of moderate severity

0% Community-acquired bronchopneumonia

0% Acute pleuritis

0% Acute lung abscess

0% Nosocomial lobar pneumonia

A 6-year-old girl complains of body temperature up to 39 C, rhinitis, dry cough, dyspnea. She

has been presenting with these signs for 5 days already. On examination her condition is of

moderate severity. Her dyspnea is of mixed genesis. Respirations are 28/min. Percussion produces

a dull sound in the right lower segments; in the same area auscultation detects weakened

respiration and fine vesicular wet crackles; coarse respiration can be detected on the left. Make the

provisional diagnosis:

Acute obstructive bronchitis

Stenosing laryngotracheitis

Acute bronchiolitis

Right-sided community acquired pneumonia*

Acute simple bronchitis


A 9-month-old child presents with fever, cough, dyspnea. The symptoms appeared 5 days ago after
a contact with a person with URTl. Objectively: the child is in grave condition. Temperature is
38oC, cyanosis of nasolabial triangle is present. rR- 54/min, nasal flaring during breathing is
observed. There was percussion dullness on the right below the scapula angle and tympanic sound
over the other areas of lungs. Auscultation revealed bilateral fine moist crackles predominating on
the right. What is the most likely diagnosis?

100% Pneumonia

0% URTI

0% Acute laryngotracheitis

0% Acute bronchitis

0% Acute bronchiolitis

An 18-month-old child was taken to a hospital on the 4-th day of the disease. The disease began
acutely with temperature 39, weakness, cough, breathlessness. He is pale, cyanotic, has had febrile
temperature for over 3 days. There are crepitative fine bubbling rales on auscultation. Percussion
sound is shortened in the right infrascapular region. X-ray picture shows non-homogeneous
segment infiltration 810 mm on the right, the intensification of lung pattern. Your diagnosis:

100% Segmentary pneumonia

0% Grippe

0% Bronchitis

0% Bronchiolitis

0% Interstitial pneumonia

4 days ago a 32-year-old patient caught a cold: he presented with sore throat, fatigue. The next
morning he felt worse, developed dry cough, body temperature rose up to 38,2oC, there appeared

muco-purulent expectoration. Percussion revealed vesicular resonance over lungs, vesicular


breathing weakened below the angle of the right scapula, fine sonorous and sibilant wheezes.
What is the most likely diagnosis?

100% Focal right-sided pneumonia

0% Bronchial asthma

0% Acute bronchitis
0% Pulmonary carcinoma

0% Pulmonary gangrene

The 10 y.o. boy has complains on headache, weakness, fever 400C, vomiting, expressed dyspnea,
pale skin with flush on right cheek, lag of right hemithorax respiratory movement, dullness on
percussion over low lobe of right lung, weakness of vesicular respiration in this zone. The
abdomen is painless and soft at palpati- on. Which disease lead to these symptoms and signs?

100% Pneumonia croupousa

0% Intestinal infection

0% Acute appendicitis

0% Acute cholecystitis

0% Flu

The 7 m.o. infant is suffering from acute pneumonia which was complicated by cardiovascular
insufficiency and respiratory failure of II degree. The accompanied diagnosis is malnutrition of II
degree. Choose the best variant of therapy:

100% Ampiox and Amicacin

0% Macropen and Penicillin

0% Penicillin and Ampiox

0% Gentamycin and Macropen

0% Ampiox and Polymixin

A 56 y.o. woman has an acute onset of fever up to 390C with chills, cough, and pain on respiration
in the right side of her chest. On physical examination: HR-90/min, BP- 95/60 mm Hg, RR-
26/min. There is dullness over the right lung on percussion. On X-ray: infiltrate in the right
middle lobe of the lung. What is the diagnosis?

100% Community-acquired lobar pneumonia with moderate severity

0% Community-acquired bronchopneumonia

0% Acute pleurisy

0% Acute lung abscess

0% Hospital-acquired lobar pneumonia


After an overexposure to cold, a 32-year-old patient developed general weakness, excessive
sweating, fever, and cough. His overall condition is moderately severe, his skin is moist, his lips are
cyanotic. His chest is symmetrically involved in the act of breathing. Under the shoulder blade on
the right, the percussion sound is slightly dull; vesicular respiration there is weakend, with
sonorous moist finely-vesicular crackles. Above the rest of the lung surface there is harsh
breathing with scattered dry crackles. The heart sounds are intensified, the second heart sound is
accentuated over the pulmonary artery. In the blood: leukocytes – 13.2 109/L, ESR – 21 mm/hour.
Make the right diagnosis:

Right-sided focal pneumonia*

Lung cancer

Tuberculosis

Acute bronchitis

Exacerbation of chronic non-obstructive bronchitis

A 40-year-old patient has acute onset of disease caused by overexposure to cold. Temperature has
increased up to 39oC. Foul-smelling sputum is expectorated during coughig. Various moist
crackles can be auscultated above the 3rd segment on the right. Blood test: leukocytes - 15,0 •
109/l, stab neutrophils - 12%, ESR- 52 mm/hour. On X-ray: in the 3rd segment on the right there is
a focus of shadow 3 cm in diameter, low density, with fuzzy smooth margins and a clearing in its
center. What disease is most likely in the given case?

100% Pneumonia complicated by an abscess

0% Infiltrative tuberculosis

0% Peripheral pulmonary cancer

0% Cystic echinococcosis

0% Pulmonary cyst

On the 4th day after recovering from a cold a patient was hospitalized with complaints of solitary
spittings of mucoid sputum. On the 2nd day there was a single discharge of about 250 ml of
purulent blood-streaked sputum. Objectively: the patient’s condition is moderately severe.
Respiratory rate

-28-30/min., Ps- 96/min., BP- 110/70 mm Hg. Respiration above the left lung is vesicular, weak
above the right lung. There are various moist crackles above the lower lobe and amphoric breath
near the angle of scapula. What is the most likely diagnosis?
100% Acute pulmonary abscess

0% Exudative pleuritis

0% Acute focal pneumonia

0% Pleural empyema

0% Pyopneumothorax

A 56 y.o. woman has an acute onset of fever up to 390C with chills, cough, and pain on respiration
in the right side of her chest. On physical examination: HR-90/min, BP- 95/60 mm Hg, Ps-
26/min. There is dullness over the right lung. On X-ray: infiltrate in the right middle lobe of the
lung en palpation. What is the diagnosis?

100% Community-acquired lobar pneumonia with moderate severity

0% Community-acquired bronchopneumonia

0% Acute pleurisy

0% Acute lung abscess

0% Hospital-acquired lobar pneumonia

tuberculosis
To replace the placeholder text on this page, you can just select it all and then start typing. But
don’t do that just yet!

MCQS

A 20-year-old adolescent lives in the nidus of tuberculous infection. The tuberculine Mantoux test
with 2 TU was determined as hyperergic. What signs determine the hyperergic test of this
adolescent?
100% 6 mm papula, necrosis

0% 20 mm papula

0% 24 mm hyperemia

0% 4 mm papula

0% 12 mm hyperemia

A 30-year-old woman complains of Subfebrile body temperature that persists for the last 3 weeks,
loss of appetite and working ability, excessive sweating (especially at night), and malaise.
Objectively, her pulmonary percussion indicates no changes in the lungs, auscultation detects
crackles in the projection of the upper lobe of the right lung. X-ray shows a dense focus of
moderate intensity, 6 mm in diameter, in segment S2. Make the provisional diagnosis:

Metastatic lung cancer

Focal pulmonary tuberculosis*

Peripheral lung cancer

Community-acquired pneumonia

Pulmonary tuberculoma

Prophylactic photoroentgenography examination of a 25 year old man revealed focal shadowings


of small and medium intensity with irregular contours in the 1st and 2nd segments of the right
lung. Which clinical form can be suspected?

100% Focal

0% Disseminated

0% Miliary

0% Fibro-cavernous

0% Tuberculoma
A 42-year-old woman working at a poultry farm complains of dyspnea, thoracic pain on the left,
increased body temperature up to 38-39oC in the evening, and cough. The patient suffers from
essential hypertension. Objectively: vesicular respiration in the lungs, vesicular resonance without
alterations. X-ray of both lungs reveals numerous small low-intensity foci, 2-3 mm in size, located
in a row along the blood vessels. ESR- 32 mm/hour. What is the most likely diagnosis?

100% Acute disseminated tuberculosis

0% Infiltrative tuberculosis

0% Pulmonary carcinomatosis

0% Focal tuberculosis

0% Community-acquired pneumonia

X-ray picture of chest shows a density and an abrupt decrease in the upper lobe of the right lung.
The middle and lower lobe of the right lung exhibit significant pneumatization. The right
pulmonary hilum comes up to the dense lobe. In the upper and middle parts of the left pulmonary
field there are multiple focal shadows. In the basal region of the left pulmonary field there are
clear outlines of two annular shadows with quite thick and irregular walls. What disease is this
X-ray pattern typical for?

100% Fibro-cavernous pulmonary tuberculosis

0% Atelectasis of the right upper lobe

0% Abscessing pneumonia

0% Peripheral cancer

0% Pancoast tumour

A patient suffering from infiltrative pulmonary tuberculosis was prescribed streptomycin,


rifampicin, isoniazid, pyrazi-namide, vitamin C. One month after the beginning of the treatment
the patient started complaining of reduced hearing and tinnitus. What drug has such a side effect?

100% Streptomycin

0% Isoniazid

0% Rifampicin

0% Pyrazinamide
0% Vitamin C

A 25-year-old man was admitted to a hospital with a 2 month history of cough and fever. A chest
x-ray showed extensive left upper lobe disease with a 2 cm cavity. All three points were strongly
positive on direct smear and grew M. Tuberculosis, fully sensitive to all first-line drugs. Patient
must be treated with:

100% Isoniazid + rifampicin + pyrazinamide

0% Streptomycin + isoniazid

0% Isoniazid + ethambutol

0% Kanamycin + ethambutol + pyrazinami-de

0% P-aminosalicylic acid + streptomycin

A 5-year-old boy was progressively getting worse compared to the previous 2 months. A chest x-
ray has shown right middle lobe collapse. A tuberculin skin test was strongly positive. What is the
most characteristic finding in primary tuberculosis?

100% Hilar or paratracheal lymph node enlargement

0% Atelectasis with obstructive pneumonia

0% Cavity formation

0% Miliary tuberculosis

0% Hematogenous dissemination leading to extrapulmonary tuberculosis

Fluorography of a 45 y.o. man revealed some foci of small intensity with nondistinct outlines on
the top of his right lung. The patient doesn’t feel worse. He has been smoking for many years.
Objectively: vesicular resonance over lungs, respiration is vesicular, rales are absent. Blood count is
not changed. What is the most probable diagnosis?

100% Focal pulmonary tuberculosis

0% Peripheral cancer of lung

0% Eosinophilic pneumonia
0% Bronchiolitis

0% Disseminated pulmonary tuberculosis

A 25-year-old patient complains of general weakness, dry cough, sweating, subfebrile temperature.
Objectively: lung auscultation reveals vesicular resiration with no wheezing. Fluorogram shows
focal shadows of high intensity in the 12 segments of the right lung. Mantoux test gave a reaction
of 16 mm of induration. What clinical form of tuberculosis is most likely?

100% Focal

0% Infiltrative

0% Disseminated

0% Tuberculoma

0% Miliary

A 30-year-old male patient had been admitted to the TB hospital because of the following changes
detected by fluorography: an ill-defined shadow of low intensity up to 1 cm in diameter in the S1 of
the right lung. CT scan showed a destruction area in the center of the shadow. Sputum analysis
revealed MTB. The patient was diagnosed with focal tuberculosis. What phases of tuberculosis are
the identified changes typical for?

100% Infiltration and disintegration

0% Infiltration and dissemination

0% Resorption and scarring

0% Disintegration and dissemination

0% Calcification and resorption

Kartagener syndrome
To replace the placeholder text on this page, you can just select it all and then start typing. But
don’t do that just yet!
MCQS

A child was born at a gestational age of 34 weeks in grave condition. The leading symptoms were
respiratory distress symptoms, namely sonorous and prolonged expiration, involving additional
muscles into respiratory process. The Silverman score at birth was 0 points, in 3 hours it was 3
points with clinical findings. Which diagnostic study will allow to diagnose the form of
pneumopathy?

100% X-ray of chest

0% Clinical blood test

0% Determination of blood gas composition

0% Proteinogram

0% Immunoassay

A 2-year-old child has been suffering since birth from recurring inflammatory diseases of lungs,
purulent pansinusites, hearing deterioration, multiple cylindrical bronchiectases. Dextrocardia is
observed. On biopsy: ultrastructural change of ciliated epithelium. What is the basis of the given
syndrome?

100% Primary ciliary diskinesia

0% Proteoglycans insufficiency

0% Surfactant deficit

0% Muscle cells atony

0% Alpha-1-antitrypsin deficiency

A boy was born at 32 weeks of gestation. 2 hours after the birth he developed respiratory distress
(RD). The RD severity assessed by Silverman score was 5. The respiratory disorders progressed,
respiratory failure could not be eliminated by Martin-Bouyer CPAP (continuous positive

airway pressure). X-ray of lungs shows reticular and nodular pattern, air bronchogram. What is
the most likely cause of respiratory distress syndrome?
100% Hyaline membrane disease

0% Segmental atelectasis

0% Bronchopulmonary dysplasia

0% Congenital pulmonary emphysema

0% Edematous hemorrhagic syndrome

7 y.o. boy with chronic sinusitis and rercurent pulmonary infections has chest X-ray
demonstrating a right-sided cardiac silhouette. What is the most likely diagnosis?

100% Kartagener syndrome

0% Cystic fibrosis (mucoviscidosis)

0% Bronchiolitis obliterans

0% Laryngotracheomalacia

0% o-antitrypsin deficiency

Mucoviscidosis (cystic fibrosis)


To replace the placeholder text on this page, you can just select it all and then start typing. But
don’t do that just yet!

MCQS

An 8 y.o. boy complains of constant cough along with discharge of greenish sputum, dyspnea
during physical activities. At the age of 1 year and 8 months he fell ill for the first time with
bilateral pneumonia that had protracted course. Later on there were recurrences of the disease 5-6
times a year, during the remission periods there was constant productive cough. What
examination results will be the most important for making a final diagnosis?

100% Bronchography
0% Roentgenography of thorax organs

0% Bacterial inoculation of sputum

0% Bronchoscopy

0% Spirography

A 3 y.o. child with weight deffici-ency suffers from permanent moist cough. There was a history of
some pneumonias with obstruction. On examination: distended chest, dullness on percussion
over the lower parts of lungs. On auscultation: a great number of different moist rales. Level of
sweat chloride is 80 mol/L. What is the most probable diagnosis?

100% Mucoviscidosis (cystic fibrosis)

0% Bronchial asthma

0% Recurrent bronchitis

0% Bronchiectasis

0% Pulmonary hypoplasia

A 2-year-old girl has a medical history of recurrent obstructive pneumonia. In the lungs various
moist and dry crackles can be auscultated, breath sounds are diminished. Sputum is thick, viscous
and difficult to expectorate. Drumstick fingers and physical developmental retardation are
observed. What preliminary diagnosis can be made?

100% Pulmonary mucoviscidosis

0% Recurrent bronchitis

0% Bronchial asthma

0% Congenital polycystic lungs

0% Pulmonary tuberculosis

The mother of a 3-month-old child came to a family doctor with complaints of her child being
physically underdeveloped and suffering from cough attacks and dyspnea. Anamnesis: the child is
the result of the second full-term pregnancy with the risk of miscarriage (the first child died of
pulmonary pathology at the age of 4 months, according to the mother). Body mass at birth is 2500
g. Cough attacks were observed from the first days of life, twice the child was treated for
bronchitis. Considering the severity of the child’s condition the doctor made the referral for
hospitalization. What diagnosis was most likely stated in the referral?

100% Mucoviscidosis (Cystic fibrosis)

0% Acute obstructive bronchitis

0% Recurrent obstructive bronchitis

0% Pertussis

0% Acute obstructive pneumonia

A 3-year-old girl presents with pertussis-like cough with thick sputum. There have been persistent
changes in lungs since the age of 6 months when she was first diagnosed with acute pneumonia.
Chloride concentration in the perspiration is 112 mEq/l. The child has been diagnosed with
mucoviscidosis. What is the basis for autosomal recessive disease -mucoviscidosis?

100% Inadequate transport of sodium and chloride ions

0% «!-antitrypsin deficiency

0% Deposition of calcium triphosphates and carbotates in the alveoles

0% Pulmonary cysts

0% Pulmonary artery hypoplasia

Bronchiectasis
To replace the placeholder text on this page, you can just select it all and then start typing. But
don’t do that just yet!

MCQS

A 30 y.o. man presents with a history of recurrent pneumonias and a chronic cough production of
foul smelling, pi-rulent sputum, ocassionally glood tinged, which is worse in the morning and on
lying down. On physical examination, the patient appears chronically ill with clubbing of fingers,
inspiratory rales at the base of lungs posteriorly. Most likely diagnosis:

100% Bronchoectasis

0% Chronic bronchitis

0% Disseminated pulmonary tuberculosis

0% Pulmonary neoplasm

0% Chronic obstructive emphysema

A 9-year-old boy has been suffering from bronchoectasis since he was 3. Exacerbations occur quite
often, 3-4 times a year. Conservative therapy results in short periods of remi- ssion. The disease is
progressing, the child has physical retardation. The child’s skin is pale, acrocyanotic, he has
"watch glass"nail deformation. Bronchography revealed saccular bronchiectases of the lower lobe
of his right lung. What is the further treatment tactics?

100% Surgical treatment

0% Further conservative therapy

0% Physiotherapeutic treatment

0% Sanatorium-and-spa treatment

0% Tempering of the child’s organism

A 72-year-old man with pneumonia complains of marked dyspnea, chest pain, severe cough with

expectoration, temperature is 39.5-40 C, no urination for the last 24 hours. Objectively, the patient

is conscious. Respiratory rate is 36/min. Percussion sound is dull over the right lower pulmonary

lobe; on auscultation there is bronchial respiration and numerous moist crackles. Blood pressure
is

80/60 mmHg. Heart rate is 120/min. Heart sounds are muffled, there is tachycardia. What tactics

should the family doctor choose for the management for this patient?

Treatment in the day patient facility


Hospitalization into the neurology unit

Hospitalization into the pulmonary unit

Hospitalization into the intensive care unit*

Outpatient treatment

A 19-year-old young man complains of cough with expectoration of purulent sputum in the
amount of 100 mL per day, hemoptysis, dyspnea, increased body temperature up to 37.8°C, general
weakness, and weight loss. The patient’s condition lasts for 4 years. Exacerbations occur 2-3 times
a year. The patient presents with malnutrition, pale skin, cyanosis of the lips, and drumstick
(clubbed) fingers. Tympanic percussion sound in the lungs weakened respiration, and various
numerous moist crackles in the lower pulmonary segments on the left can be observed in this
patient. Complete blood count: erythrocytes — 3.2 • 1012/L, leukocytes — 8.4 • 109/L, ESR — 56
mm/hour. On X-ray: lung fields are emphysematous, and the left pulmonary root is deformed and
dilated. What is the most likely diagnosis?

Multiple bronchiectasis of the left lung*

Left-sided pulmonary cystic dysplasia

Suppuration of the cyst in the left lung

Chronic left-sided pneumonia

Chronic abscess of the left lung

A 58-year-old patient, a heavy drinker and smoker, came to a hospital with complaints of constant
coughing and shortness of breath. Lately, he has been losing weight. His cervical lymph nodes are
enlarged and dense, the tissues above them exhibit no tension. Chest X-ray has shown fibrosis of
an upper pulmonary lobe and left-sided pleurisy. The pleural fluid is straw-colored, with protein
levels of 52 g/L and a high lymphocyte count. Malignant cells were not detected. Inoculation of
the pleural fluid produced no microbial growth one week later. Make the right diagnosis:

Sarcoidosis

Atypical pneumonia

Systemic lupus erythematosus


Pulmonary tuberculosis

Bronchiectasis*

A 57-year-old man complains of cough with profuse mucopurulent sputum (up to 150 mL per 24
hours). Objectively, he has drumstick fingers with watch-glass nails. Percussion produces a
shortened sound over the lungs. Auscultation detects moderate and large bubbling crackles.
Complete blood count shows leukocytosis and a left shift in the leukogram. Chest X ray shows
intensified pulmonary pattern and ring-like shadows. Bronchography detects multiple cylindrical
thickenings of the bronchi with clear margins. Make the provisional diagnosis: empyema

Pulmonary gangrene

Pulmonary echinococcosis

Chronic pleural empyema

Bronchiectasis*

Pulmonary cyst

A 9-year-old boy has been suffering from multiple bronchiectasis since he was 3 years old.
Exacerbations occur frequently (34 times a year), after conservative therapy there are short
remission periods. The disease progresses, the child is physically underdeveloped, presents with
pale skin, acrocyanosis, deformed nail plates in the shape of ”clock-face” Bronchography reveals
saccular bronchiectases in the lower lobe of the right lung. What further treatment tactics should
be chosen?

100% Surgical intervention

0% Continuation of conservative therapy

0% Physiotherapy

0% Sanatorium-and-spa treatment

0% Physical training

A 28-year-old female patient has been admitted to a hospital. She states to be ill for 12 years. On
examination she has been diagnosed with bronchiectasis with affection of the left lower lobe of
lung. What is the optimal treatment tactics for this patient?

100% Left lower lobectomy

0% Left-sided pneumoectomy
0% Antibiotic therpy

0% Active drainage of the left pleural cavity

0% Bronchopulmonary lavage

A patient’s X-ray image (anteroposterior projection) shows deformation of lung pattern,


pneumofibrosis, reticular (honeycomb) lung pattern of lower pulmonary segments, cylindric and
fusiform lumps. The most likely diagnosis is:

100% Multiple bronchiectasis

0% Right lung abscess

0% Non-hospital-acquired pneumonia

0% Lungs maldevelopment

0% Diffuse pulmonary fibrosis

A 20 daily y.o. female patient is suffering from chronic bronchitis. Recently there has been
production about 0,5 L of purulent sputum with maximum discharge in the morning. Fingers are
like "drum sticks", there are "watching glass"nails. What is the most probable diagnosis?

100% Bronchiectasis

0% Pneumonia

0% Chronic bronchitis

0% Gangrene of lungs

0% Tuberculosis

lung cancer
To replace the placeholder text on this page, you can just select it all and then start typing. But
don’t do that just yet!
MCQS

A 67 y.o. patient complains of dyspnea, breast pain, common weakness. He has been ill for 5
months. Objectively: t0- 37, 30C, Ps- 96/min. Vocal tremor over the right lung cannot be
determined, percussion sound is dull, breathing cannot be auscultated. In sputum: blood
diffusively mixed with mucus. What is the most probable diagnosis?

100% Cancer of lung

0% Macrofocal pneumonia

0% Bronchoectatic disease

0% Focal pulmonary tuberculosis

0% Exudative pleuritis

A 56-year-old patient complains of having persistent chest pain on the right for the last 2 months.
The pain is not associated with respiration. He also complains of cough with blood-streaked
sputum, weakness, decreased performance, fatigue. Chest radiograph shows a globular shade of
4x6 cm connected to the root of the lung in the lower part of the right lung. What is the most
likely diagnosis?

100% Peripheral lung cancer

0% Metastasis

0% Lung abscess

0% Pneumonia

0% Tuberculoma

During preventive examination a 58-year-old man on chest X-ray presents with multiple globular
pale shadows 3 cm in diameter within parenchyma of the both lungs. Examination in the
oncologic hospital: the primary focus is not found; transbronchial biopsy with cytologic
investigation detected cells of glandular neoplasm. What tactics should the physician choose?

100% Polychemotherapy courses

0% Exploratory laparotomy

0% Exploratory thoracotomy

0% Laparoscopy
0% Symptomatic treatment at home

A 29-year-old patient works as a motor mechanic. Anamnesis shows frequent exposure to cold,
exacerbation of chronic bronchitis attended by cough with relati-vly small amount of
mucopurulent sputum, subfebrility, sometimes joined by hemoptysis and pain in the right side of
chest. Breathing is vesicular. X-ray shows shadows and sharp decrease in size of the lower lobe
distinctly visible on the X-ray image as a streak 2-3 cm wide situated at the angle from lung root to
the frontal costodiaphragmatic recess. The most likely diagnosis is:

100% Peripheral lung cancer

0% Bronchiectasis

0% Pneumonia

0% Middle lobe syndrome

0% Interlobular pleurisy

A 63-year-old male patient complains of cough with expectoration of mucous blood-streaked


sputum, asthma, low-grade fever, general weakness. These presentations have been observed for 3
months. The patient has been a smoker since childhood. Objectively: to- 37,4oC, respiratory rate is
26/min, Ps- 82/min, rhythmic. AP- 130/85 mm Hg. There is limited breathing movement in the
right side of chest cavity, as well as percussive dullness and diminished breath sounds. Radiograph
shows a homogeneous opacity of the pulmonary field on the right with the mediastinum
displacement to the affected side. What is the most likely diagnosis?

100% Central lung cancer

0% Pleural effusion

0% Pleuropneumonia

0% Pulmonary tuberculosis

0% Bronchiectasis

A 57-year-old man, a miner, complaints of a pain in his chest, dyspnea on physical exertion,

excessive sweating, constant sub febrile temperature, and cough that produces blood-steaked

sputum. He has been smoking for approximately 40 years (2 packs a day) and frequently has

”pneumonias”. Survey chest X-ray shows a triangular shadow in the middle lobe of the right lung.
One of the apices of the shadow points to the lung root. Cardiac and mediastinal shadows are

displaced toward the affected area. Make the provisional diagnosis:

Tuberculosis of the right lung

Pneumoconiosis

Right-sided pleuropneumonia

Cancer of the right lung*

Chronic bronchitis

A 55 y.o. male patient complains of weakness during 2 months, pain in the right side of the thorax,
cough, blood-streaked sputum. On X-ray: intensive triangle shadow in the area of lower lobe that
is connected to mediastinum. What is the most likely disorder in the lungs?

100% Central cancer of lungs

0% Tuberculosis of lungs

0% Bronchiectasia

0% Pulmonary infarction

0% Pleuropneumonia

A 67 y.o. patient complains of dyspnea, breast pain, common weakness. He has been ill for 5
months. Objectively: t0- 37, 30C, Ps- 96/min. Vocal tremor over the right lung cannot be
determined, percussion sound is dull, breathing cannot be auscultated. In sputum: blood
diffusively mixed with mucus. What is the most probable diagnosis?

100% Lung cancer

0% Macrofocal pneumonia

0% Bronchoectatic disease

0% Focal pulmonary tuberculosis

0% Exudative pleuritis
LUNG GANGRENE
To replace the placeholder text on this page, you can just select it all and then start typing. But
don’t do that just yet!

MCQS

A 52 year old patient complains about pain in the right part of her chest, dyspnea, cough with a lot
of foul-smelling albuminoid sputum in form of "meat slops". Objectively: the patient’s condition is
grave, cyanosis is present, breathing rate is 31/min, percussion sound above the right lung is
shortened, auscultation revealed different rales. What is the most probable diagnosis?

100% Lung gangrene

0% Lung abscess

0% Pleura empyema

0% Multiple bronchiectasis

0% Chronic pneumonia

A patient complains of a tormental (agonizing) cough with expectoration of up to 600 ml/daily


purulent chocolatecolor sputum with a decay smell. Onset of illness was abrupt, t0- 390C, fever of
irregular type. There is the area of darkening with a cavity in a center on X-ray film, with irregular
contours and level of liquid. What disease is the question?

100% Gangrene of lung

0% Tuberculosis

0% Bronchiectatic illness

0% Pneumonia complicated by an abscess

0% Lobar pneumonia
ASTHMA
To replace the placeholder text on this page, you can just select it all and then start typing. But
don’t do that just yet!

MCQS

A 9 year old girl with a history of intermittent wheezing for several years is brought to the
pediatrician. The child has been taking no medications for some time. Physical examination
reveals agitation and perioral cyanosis. Intercostal and suprasternal retractions are present. The
breath sounds are quiet, and wheezing is audible bilaterally. The child is admitted to the hospital.
Appropriate interventions might include all of the following EXCEPT:

100% Prescribe nebulized cromolyn sodium

0% Prescribe intravenous aminophylline

0% Administer supplemental oxygen

0% Prescribe intravenous corticosteroids

0% Prescribe nebulized metaproterenol

A 27-year-old patient with a history of ronchial asthma was stung by a bee. He had a sensation of
chest compression, breath shortage, difficult expiration, sense of heat in the upper half of body,
dizziness, apparent itch, convulsions. Objectively: noisy wheezing breath, AP - 90/60 mm Hg, Ps-
110 bpm. Auscultation revealed weak rhythmic heart sounds, rough respiration above lungs,
sibilant rales. What drug group should be administered in the first place?

100% Glucocorticoids

0% Methylxanthines

0% Cardiac glycosides

0% Anticonvulsive

0% Analgetics

A 20 y.o. patient with bronchial asthma experiences dyspnea attacks 3-4 times a week. Nocturnal
attacks are 1 time a week. FEV1- 50% of necessary figures, during the day it’s variations is 25 %.
What is the severity of bronchial asthma condition?
100% Moderate severity condition

0% Mild condition

0% Serious condition

0% Asthmatic status

0% Intermittent flow

A 13-year-old boy is complaining of a dry cough and shortness of breath. The onset of the disease
was one year ago. He has brief asphyxia attacks that occur 1–2 times per month. Objectively, the
child is anxious and has expiratory dyspnea, his skin is pale, his nasolabial triangle is cyanotic. His
respiratory rate is 48/min. Percussion produces a banbox resonance over the lungs; auscultation
detects weakened breathing with bilateral dry wheezing. Forced expiratory volume is 80% of the
normal. What medicine should be prescribed to this boy?

Euphyllin (Aminophylline)

Indomethacin

Salbutamol*

Prednisolone

Suprastin (Chloropyramine)

A 7-year-old boy after playing with a cat suddenly developed problems with breathing. Objectively,
the boy is pale, frightened, and sits, leaning onto his arms. His body temperature is 36.6°C, heart
rate – 120/min., respiration rate – 42/min., speaks in syllables. The exhale is long and accompanied
by wheezing. Percussion detects a bandbox resonance over the lungs. An inhalation of ventolin
(salbutamol) solution was started via a nebulizer. What can help assess the child’s need for oxygen
therapy in this case?

Pulse oximetry*

Assessment of accessory respiratory muscles involvement

Chest X-ray

Pneumotachometry

Spirometry
A 10-year-old patient has a history of mild bronchial asthma. During a regular check-up the
patient should be recommended:

100% To avoid allergenic food

0% To avoid body tempering procedures

0% To avoid sports

0% To avoid spa treatment

0% To avoid going to the seaside

A 36-year-old female has a 7-year history of pollen allergy. Over the last 2 years in August and
September (during ragweed flowering), the patient has had 2-3 asthma attacks that could be
treated with one dose of salbutamol. Objectively: body temperature - 36,5oC, respiratory rate -
18/min, Ps- 78/min, AP-115/70 mm Hg. There is vesicular breathing above the lungs. Cardiac
sounds are sonorous, of regular rhythm. What drug would be most effective to prevent asthma
attacks during the critical season for the patient?

100% Intalum inhalation

0% Berotec inhalation

0% Atrovent inhalation

0% Suprastin administration

0% Theopecum administration

A woman with atopic bronchial asthma was found to have one allergen to dog hair

+++. Carpets were removed from the apartment, the apartment was renovated, and the

air conditioner was installed. However, recurrent asphyxia attacks still occur every

night, despite the patient undergoing pathogenetic therapy. What long-term treatment

tactics can help this patient to decrease her sensitivity to the allergen?

Continuation of prior treatment

Referral for speleotherapy


*Specific hyposensitization

Buteyko breathing technique

Antihistamine therapy

A 10-year-old boy was brought to the hospital with complaints of expiratory

dyspnea, respirations are 30/min. He explains his state by a change in the weather

conditions For the last 4 years the boy has been registered for regular check-ups due

to his diagnosis of third degree persistent bronchial asthma. To provide emergency aid

for this child, first he needs to be given:

Dexamethasone

Salbutamol or short-acting B2-agonists*

Euphylline (Aminophylline)

Adrenaline

Claritin (Loratadine)

A hospital admitted an 11-year-old boy diagnosed with medium-severe asthma, exacerbation


period. In order to arrest the attacks the boy was administered broncholytic nebulizer therapy.
During the day the child’s condition stabilized. What is the most appropriate method for further
monitoring of respiratory function in this patient?

100% Peak flowmetry

0% Spirometry

0% Pneumotachometry

0% Bronchodilatation tests

0% Veloergometry
A 38 y.o. woman is seriously ill. She complains of frequent paroxysms of expiratory dyspnea. The
last paroxysm lasted over 12 hours and failed to respond to theophylline. The skin is palish gray,
moist, RR of 26/min. On auscultation, breath sounds are absent over some areas. Your preliminary
diagnosis?

100% Bronchial asthma, status asthmaticus

0% Chronic obstructive bronchitis

0% Atopic bronchial asthma, respiratory failure of the III degree

0% Bronchiectasis, respiratory failure of the II-III degree

0% Ischemic heart disease, pulmonary edema

A boy of 7 y.o. had an attack of asthma and distant whistling rales after playing with a dog. In the
medical hystory: atopic dermatitis caused by eating eggs, chicken, beef. What group of allergens is
the reason of the development of bronchial astma attacks?

100% Epidermal

0% Dust

0% Pollen

0% Itch mite

0% Chemical

A 6 y.o. asthmatic child was taken to the emergency hospital because of severe coughing and
wheezing for the last 24 hours. Physical examination reveals that the child is excitable, has
intercostal and suprasternal retractions, expiratory wheezing throughout all lung fields,
RR-60/min. Initial treatment may include the prescription of:

100% Subcutaneous epinephrine

0% Parenteral phenobarbital

0% Intravenous fluids in the first 2 h to compensate water deficiency

0% N-acetyl cysteine and cromolyn by inhalation

0% Parenteral gentamicyn
A 49-year-old patient complains of dyspnea, cough. There are no sputum discharges. He has
repeatedly used salbutamol and intal but with no effect. Objectively: he is only able to sit while
leaning on the table. Cyanosis of face, acrocyanosis are present. Breathing is shallow, laboured, in
some parts it cannot be auscultated; there are diffuse rales, expiration is significantly prolonged.
Heart sounds are muffled, tachycardia is present. Ps - 112/min., AP - 110/70 mm Hg. Liver is located
near the costal arch. There are no peripheral edemata. What is your provisional diagnosis?

100% Status asthmaticus

0% Chronic obstructive bronchitis

0% Bronchiale asthma, moderate gravity

0% Foreign object aspiration

0% Cardiac asthma

A 23-year-old patient had taken 1 g of aspirin to treat acute respiratory infection. After that he
developed an asthmatic fit with labored expiration that was arrested by introduction of
aminophylline. The patient’s medical history is not burdened with allergies. The patient has
undergone two surgeries for nasal polyposis in the past. What diagnosis is most likely?

100% Aspirin-induced asthma

0% Atopic bronchial asthma

0% Infectious allergic bronchial asthma

0% Exercise-induced asthma

0% Symptomatic bronchospasm

A 56-year-old woman was diagnosed with stage 2 hypertension of the 2nd degree. She belongs to
the group of moderate risk and has bronchial asthma. What group of drugs is contraindicated to
this patient?

β-blockers*

Diuretics

Angiotensin-converting enzyme inhibitors

Imidazoline receptor antagonists

Calcium antagonists
A 7-year-old child was brought to a doctor for a check. The child has a 4-year history of bronchial
asthma, asthma attacks occur mainly in spring and summer. Allergy tests revealed hypersensitivity
to poplar seed tufts, field herbs. What recommendation should be given?

100% Specific hyposensitization

0% Physiotherapy

0% Treatment at a health resort

0% Phytotherapy

0% Needle reflexotherapy

PNEUMOCONIOSIS
To replace the placeholder text on this page, you can just select it all and then start typing. But
don’t do that just yet!

MCQS

A patient, aged 52, works as a street cleaner. He has been suffering from pneumoconiosis for 2
years. What treatment should be recommended?

100% Alkaline inhalations

0% Broncholitics

0% Oil inhalations

0% Sulfonilamides

0% Antibiotics

A survey radiograph of a miner (24 years of service record, the dust concentration in the
workplace is at the rate of 260-280 mg/m3 with 15% of free silica) shows lung changes that are
typical for pneumoconiosis. What type of pneumoconiosis is it?
100% Anthracosilicosis

0% Carboconiosis

0% Silicatosis

0% Anthracosilicatosis

0% Silicosis

A patient is 50 years old, works as a builder with 20 years of service record. He was admitted to the
hospital for chest pain, dry cough, minor dyspnea. Objectively: sallow skin, acrocyanosis, asbestos
warts on the hands. In lungs - rough respiration, diffuse dry rales. The x-ray picture shows
intensification of pulmonary pattern, signs of pulmonary emphysema. What is the most likely
diagnosis?

100% Asbestosis

0% Lung cancer

0% Pneumonia

0% Chronic obstructive bronchitis

0% Tuberculosis

A worker at a porcelain factory who has been in service for 10 years complains of cough, dyspnea,
ache in his chest. What occupational disease are these complaints most typical for?

100% Silicosis

0% Multiple bronchiectasis

0% Chronic dust bronchitis

0% Occupational bronchial asthma

0% Chronic cor pulmonale

A 38-year-old woman works in flax processing, she dries fax. She came to the

hospital complaining of difficult breathing, constricting sensation in her chest, and


cough attacks. These signs appear on the first day of her working week and gradually

diminish on the following days. What respiratory disease is likely in this case?

Allergic rhinopharyngitis

Asthmatic bronchitis

Bronchial asthma

Byssinosis*

Silicosis

The man, aged 42, applied to the therapeutist with complaints of pricking pains in scapulas area,
dyspnea on physical exertion, cough with discharge of small amount of sputum. During 10 years
he works in coal mining. On percussion-box-note sound in the lower parts, on auscultation- a
harsh breathing. There were no changes in the heart. Possible diagnosis?

100% Silicosis

0% Tuberculosis of lungs

0% Silicatosis

0% Bronchiectatic disease

0% Chronic bronchitis

A 38 year old man worked at roofing and drain pipes production for 15 years. He seeks medical
help for expiratory breathlessness on exertion, and dry cough. On exam, wheezes above both
lungs, grayish warts on fingers are seen. Factory physician has diagnosed asbestosis. What method
is the most important for this diagnosis?

100% Chest X-ray

0% Bronchoscopy

0% Blood gas analysis

0% Spirography

0% Electrocardiography
Chronic obstructive lung disease
To replace the placeholder text on this page, you can just select it all and then start typing. But
don’t do that just yet!

MCQS

A 52 y.o. hard smoker complains of persistent cough with purulent sputum discharge especially in
the mornings, dyspnea provoked even by slight physical exercises, wheezing chest, tachypnoe,
general weakness. He considers himself to be ill for 12 years. The foresaid presentations appear 3-4
times per year usually after a common cold and have tendency to progress. What disease do you
think about first of all?

100% Chronic obstructive lung disease

0% Bronchial asthma

0% Mucoviscidosis (cystic fibrosis)

0% Bronchoectatic disease

0% Aspergillosis

Topographic percussion of lungs in a patient who got a serious job-related barotrauma revealed
that the lower lungs borders were located one rib below normal, there was a significant increase in
both lungs height and Kronig’s isthmus. What disease should be suspected in the first place?

100% Pulmonary emphysema

0% Exudative pleuritis

0% Chronic bronchitis

0% Bronchial asthma

0% Pneumothorax

A 52 y.o. patient complains of dyspnea caused even by moderate physical activity, cough with
hardly secreted sputum. He has been ill for 12 years. Objectively: BR- 26/min. Lung examination:
tympanitis, diminished vesicular breathing with prolonged expiration, disseminated dry rales. In
the past he was taking only theopecym or ami-nophylline inravenously. Prescribe him the basic
treatment after the exacerbation is suppressed:

100% Atrovent

0% Alupent

0% Inhacort

0% Tilade

0% Aminophylline

Allergic rhinitis
To replace the placeholder text on this page, you can just select it all and then start typing. But
don’t do that just yet!

MCQS

A 10-year-old boy came to the polyclinic with complaints of stuffy nose. It is known that these
signs occur in the child periodically (in spring and autumn). He has a history of atopic dermatitis.
The father of the child has bronchial asthma. Objectively, the boy’s face is pale and slightly
swollen. Respirations are 22/min. Auscultation detects vesicular respiration over the lungs.
Rhinoscopy shows swollen and pale nasal mucosa. What disease can be suspected?

Recurrent respiratory disease

Acute maxillary sinusitis

Acute adenoiditis

Acute rhinitis

Allergic rhinitis*
Pulmonary edema
To replace the placeholder text on this page, you can just select it all and then start typing. But
don’t do that just yet!

MCQS

A 54-year-old drowned man was rescued from the water and delivered to the shore. Objectively:
the man is unconscious, pale, breathing cannot be auscultated, pulse is thready. Resuscitation
measures allowed to save the patient. What complications may develop in the near future?

100% Pulmonary edema

0% Respiratory arrest

0% Encephalopathy

0% Cardiac arrest

0% Bronchospasm

A 65-year-old man complains of asphyxia, cough with pink foaming sputum, sensation of lack of
air, and fear of death. He has orthopnea, pale skin, and acrocyanosis and is covered in cold sticky
sweat. His respiration is harsh, in the lower posterior segments there are wet finely vesicular and
moderately vesicular crackles on both sides. His respiratory rate is 40/min. The heart sounds are
markedly muffled. At the cardiac apex, the gallop rhythm is observed. Make the right diagnosis:

Infarction pneumonia

Pulmonary edema*

Status asthmaticus

Pulmonary embolism

Croupous pneumonia

A 67-year-old female patient with hypertensive crisis has asthma, cough with expectoration of
frothy pink sputum, moist rales in the lungs. The patient stays in sitting position, respiratory rate
is 40/min, AP-214/136 mm Hg, heart rate - 102/min. What is the most rational tactics of this
patient management?
100% Intravenous administration of furosemide

0% Urgent pneumography

0% Bed rest, lying position

0% Intravenous administration of a ^-blocker

0% Tactics can be determined after ECG and chest radiography

A 58-year-old patient complains of a headache in the occipital region, nausea, choking,


opplotentes. The presentations appeared after a physical exertion. Objectively: the patient is
excited. Face is hyperemic. Skin is pale. Heart sounds are regular, the 2nd aortic sound is
accentuated. AP- 240/120 mm Hg, HR-92/min. Auscultation reveals some fine moist rales in the
lower parts of the lungs. Liver is not enlarged. ECG shows signs of hypertrophy and left ventricular
overload. What is the most likely diagnosis?

100% Complicated hypertensic crisis, pulmonary edema

0% Acute myocardial infarction, pulmonary edema

0% Bronchial asthma exacerbation

0% Uncomplicated hypertensic crisis

0% Community-acquired pneumonia

Airway foreign body


To replace the placeholder text on this page, you can just select it all and then start typing. But
don’t do that just yet!

MCQS

A 5-year-old child was brought to the ENT department by an ambulance. The child
presents with cough and difficult respiration From the patient’s history it is known that

the child was playing with a toy construction set, when suddenly started coughing and

developed labored breathing Examination detects periodical cough, labored

expiration, and respiratory lag in the left side of the child’s thorax. Auscultation:

diminished respiration on the left. Percussion: tympanitis. X-ray shows a displacement

of the mediastinal organs to the right. Make the diagnosis:

A foreign body in the right bronchus, partial bronchostenosis

A foreign body in the left bronchus, valvular bronchostenosis*

A foreign body in the right bronchus, valvular bronchostenosis

A foreign body in the trachea

A foreign body in the left bronchus, complete bronchostenosis

A 3-year-old child, while playing. suddenly developed cough attacks and problems with breathing.
Objectively, the child has a dry cough and mixed type dyspnea. Auscultation detects a small
amount of dry crackles in the lungs. Respiration is weakened on the right. The child does not
attend kindergarden and has all necessary immunizations for that age. What pathology can be
suspected?

Airway foreign body*

Pertussis

Acute respiratory viral infection

Bronchial asthma

Pneumonia

When playing in a kindergarten a 3-year-old child sudenly developed dyspnea, paroxysmal


compulsive dry cough. The face is cyanotic, the eyes are tearful. Vomiting occurred several times.
Breathing is weakened over the whole right side of the chest. The provisional diagnosis is:

100% Foreign body

0% Obstructive bronchitis

0% Bronchial asthma

0% Hysteria fit

0% Stenosing laryngotracheitis

A hospital admitted a patient with coarse breathing (obstructed inspiration), skin cyanosis,
tachycardia and arterial hypertension. He has a histrory of bronchial asthma. An hour ago he was
having salbutamol inhalation and forgot to remove a cap that was aspired while taking a deep
breath. What measures should the doctor take?

100% To perform the Heimlich manoever

0% To perform conicotomy immediately

0% To send for an anesthesiologist and wait for him

0% To use an inhalation of ^-adrenoceptor agonist

0% To make a subcutaneous injection of dexamethasone

A 3-year-old child was playing in a playpen when he suddenly developed paroxysmal cough and
shortness of breath. Objectively: dry cough, mixed dyspnea. Lung auscultation revealed some
wheezes. Breathing sounds on the right are diminished. The child doesn’t mix with other children.
Immunization is age-appropriate. What pathological condition can be suspected?

100% Foreign body in the respiratory tracts

0% Pneumonia

0% Acute respiratory viral infection

0% Pertussis

0% Bronchial asthma
Somatoform autonomic dysfunction
To replace the placeholder text on this page, you can just select it all and then start typing. But
don’t do that just yet!

MCQS

A 15-year-old girl complains of dizziness and sensation of lack of air that she

develops in emotionally straining situations. Relief occurs after she takes corvalol.

Objectively hyperhidrosis and marble-like pattern of the skin of her palms and feet.

Clinical and instrumental examination revealed no organic changes in the central

nervous, cardiovascular, and respiratory systems. What provisional diagnosis can be

made?

Stenosing laryngotracheitis

Obstructive bronchitis

Bronchial asthma

Acute epiglottitis

Somatoform autonomic dysfunction*

PULMONARY EMBOLISM
To replace the placeholder text on this page, you can just select it all and then start typing. But
don’t do that just yet!
MCQS

After significant physical exertion a 66-year-old man with deep vein thrombosis of the extremities
developed shortness of breath, intense pain in the chest on the left, marked palpitations. The
patient’s condition is grave, his face is cyanotic, the cervical veins are swollen, BP is 60/40 mm Hg.
What investigation method would be the most advisable in this case?

100% Selective angiopneumography

0% Chest X-ray

0% Echocardiography

0% Magnetic resonance imaging of the chest

0% Fiber-optic bronchoscopy

A 55-year-old male had been treated at the surgical department for acute lower-extremity
thrombophlebitis. On the 7th day of treatment he suddenly developed pain in the left part of
chest, dyspnea and cough. Body temperature was 36,1oC, respiratory rate - 36/min. The patient
was also found to have diminished breath sounds without wheezing. Ps- 140/min, thready. AP-
70/50 mm Hg. The ECG shows QiII^ syndrome. What is the most likely diagnosis?

100% Pulmonary embolism

0% Myocardial infarction

0% Cardiac asthma

0% Bronchial asthma

0% Pneumothorax

A 45-year old man with thrombophlebitis of the deep veins in his legs suddenly after

physical exertion developed sharp pain in his thorax on the right, dyspnea, and hemoptysis.

Objectively his condition is severe; he presents with acrocyanosis, shortening of pulmonary

percussion sound on the right, and weakened respiration. Respiration is 30/min, blood pressure is

110/80mm Hg. ECG shows sinus tachycardia, his heart rate is 120/min, the electrical axis of the

heart deviates to the right. S1-Q3. What is the most likely diagnosis?
Community acquires right sided pneumonia

Right-sided exudative pleurisy

Spontaneous pneumothorax

Pulmonary embolism*

Cancer of the right lung

DIAGNOSTICS IN PNEUMOLOGY
To replace the placeholder text on this page, you can just select it all and then start typing. But
don’t do that just yet!

MCQS

A child was born at 34 weeks of gestation in bad condition. The cardinal symptoms show
respiratoty disorders: sound prolonged expiration, additional muscles taking part in breathing,
crepitation rales on the background of the rough breath sounds. Assesment according to
Silverman’s scale was 0, in 3 hours- 6 with presence of clinical data. What diagnostic method can
determine pneumopathy’s type in the child?

100% Chest X-ray

0% Blood test

0% Blood gases

0% Proteinogram

0% Immunologic investigation

A 25-year-old victim of a road accient complains of chest pain, dyspnea. Objectively: the patient is
in a grave condition, Ps- 120/min, AP- 90/70 mm Hg.There is pathological mobility of fragments of
III-V ribs on the right. Percussion reveals a box sound over the right lung, breathing sounds
cannot be auscultated on the right. What examination should be administered in the first place?

100% X-ray of chest organs

0% Bronchoscopy

0% Pleural puncture

0% USI of chest organs

0% Thoracoscopy

A 15 y.o. girl was examined. Her medi- cal history registers gradual onset of fever, malaise, loss of
weight. There was nothing typical about the kind of fever which has been present for more than
7-10 days and changed quickly. Physical examination didn’t give evident results. What is the only
most important examination for excluding miliary tuberculosis?

100% Chest X-ray

0% Liver or bone marrow biopsy

0% Tuberculin skin testing

0% Sputum smear and culture of m. tuberculosis

0% Bronchoscopy

A worker, aged 38, working in the slate production during 15 years, complains of expiratory
exertional dyspnea, dry cough. On examination: deafening of the percutory sounds in
interscapular region, rough breath sounds, dry disseminated rales. On fingers’ skin - greyish warts.
Factory’s sectorial doctor suspects asbestosis. Which method is the most informative for diagnosis
verification?

100% Thorax roentgenography

0% Bronchoscopy

0% Spirography

0% Bronchoalveolar lavage

0% Blood gases examination


A 47-year-old female patient complains of cough with purulent sputum, pain in the lower left
chest, periodical body temperature rise. She has been suffering from this condition for about 10
years. Objectively: "drumstick"distal phalanges. What examination would be the most informative
for making a diagnosis?

100% Bronchography

0% Bronchoscopy

0% Survey radiograph of lungs

0% Pleural puncture

0% Bacteriological analysis of sputum

A 52-year-old male patient complains about attacks of asphyxia, pain in his right side during
respiration. These manifestations turned up all of a sudden. It is known from his anamnesis that
he had been treated for thrombophlebitis of the right leg for the last month. In the admission
ward the patient suddenly lost consciousness, there was a sudden attack of asphyxia and pain in
his side. Objectively: heart rate - 102/min, respiratory rate - 28/min, AP- 90/70 mm Hg.
Auscultation revealed diastolic shock above the pulmonary artery, gallop rhythm, small bubbling
rales above the lungs under the scapula on the right, pleural friction rub. What examination
method will be the most informative for a diagnosis?

100% Angiography of pulmonary vessels

0% Echocardioscopy

0% Study of external respiration function

0% ECG

0% Coagulogram

A 72-year-old man has been brought into a vascular surgery department with complaints of pain
and chills in his legs. Ultrasound of his leg arteries shows atherosclerotic lesions of the vascular
wall. What diagnostic method is necessary to determine the localization and the extent of the
pathologic process in this case?

Thermometry

Angiography*

ECG
X-ray of the extremities

Chest X-ray

A 9-year-old girl has been admitted to a hospital for an elevated body temperature (39,8°C),
painful dry cough, abdominal pain on the right. Examination reveals dullness on percussion on
the right, diminished breath sounds, crepitus. What study is required to make a diagnosis?

100% Radiography of the chest cavity

0% USI of the chest cavity

0% Pleural puncture

0% Bronchoscopy

0% Bronhography

As a result of lifting a load a 62-year-old female felt acute pain in the lumbar region, in a buttock,
posterolateral surface of her right thigh, external surface of the right shin and dorsal surface of
foot. Objectively: weakness of the anterior ti-bial muscle, long extensor muscle of the right toes,
short extensor muscle of the right toes. Low Achilles reflex on the right. Positive Lasegue’s sign.
What examination method would be the most effective for specification of the diagnosis of
discogenic compression of L5 root?

100% Magnetic resonance scan

0% Spinal column X-ray

0% Electromyography

0% Angiography

0% Lumbar puncture

As a result of load lifting a 68-year-old woman developed acute pain in the lumbar region, in a
buttock, posterolateral surface of her right thigh, external surface of the right shin and dorsal
surface of foot. Objectively: weakness of the anterior tibial muscle, long extensor muscle of the
right toes, short extensor muscle of the right toes. Low Achilles reflex on the right. Positive
Lasegue’s sign. What examination method would be most effective for specification of the
diagnosis of L5 root discogenic compression?

100% Magnetic resonance scan

0% Spinal column X-ray


0% Electromyography

0% Angiography

0% Lumbar puncture

A 35 year old female patient suffering from cholelithiasis has broken her diet, and this caused an
acute pain attack in the right subcostal are. The pain eased off on the third day, but the patient got
progressing jaundice. What non-invasive diagnostic method should be applied?

100% Endoscopic retrograde cholangiopancreatography

0% Infusive cholecystocholangiography

0% Test for bilirubin

0% Duodenal probing

0% Survey radiography of abdominal organs

A 52 year old man has recurrent transient ischemic attacks. Auscultation of the carotid arteries
detected murmur. What diagnostic method is to be applied in the first place?

100% Ultrasound dopplerography

0% CT of the brain

0% MRI of the brain

0% Cerebral angiography

0% Electroencephalography

A 32 y.o. patient lives in an endemic echinococcous region. For the last 6 months he has been
having pain in the right hypochondrium, temperature rise. An echinococcus liver affection is
suspected. What type of examination will be the most informative in this case?

100% USI examination

0% Plan radiography of abdominal cavity

0% Biochemical laboratory analysis

0% Angiography
0% Liver scanning

A 32-year-old patient lives in an area endemic for echinococcosis. In the last 6 months he reports
of pain in the right subcostal area, fever. He is suspected to have liver echinococcosis. What study
would be the most informative in this case?

100% USI

0% Survey radiography of abdominal cavity

0% Biochemical laboratory examination

0% Angiography

0% Liver scanning

A 38-year-old man lives in an area that is endemic for echinococcosis. For the last 6 months he has
been suffering from pain in his right subcostal region and fever. Echinococcal liver damage was
suspected. What study will be the most informative in this case?

Liver scan

Ultrasound examination*

Biochemical testing

Angiography

Survey X-ray-of the abdominal cavity

A patient after a blow to the head developed general symptoms of cerebral disturbance, nausea,
vomiting, focal signs -hemi-hyperreflexia S>D, hemihyperesthesia on the left, marked meningeal
syndrome. Neither cranial X-ray nor computer tomography revealed any pathologies. What
examination method would allow making and clarification of the diagnosis?

100% Lumbar puncture

0% Echoencephalography

0% Electroencephalography

0% Angiography
0% Pneumoencephalography

A 30-year-old patient, who has been suffering from headaches, suddenly developed extreme
headache after lifting a heavy load, as if he had been hit over the head. Nausea, vomiting, and
slight dizziness are observed. In a day he developed pronounced meningeal syndrome and body
temperature up to 37,6oC. A doctor suspects subarachnoid hemorrhage. What additional
examination is necessary to confirm this diagnosis?

100% Lumbar puncture with investigation of the spinal fluid

0% Skull X-ray

0% Computed tomography of the brain

0% Rheoencephalography

0% Angiography of the brain vessels

plague
To replace the placeholder text on this page, you can just select it all and then start typing. But
don’t do that just yet!

MCQS

A patient has got pain in the axillary area, rise of temperature developed 10 hours ago. On
examination: shaky gait is evident, the tongue is coated with white deposit. The pulse is frequent.
The painful lymphatic nodes are revealed in the axillary area. The skin over the lymph nodes is
erythematous and glistering. What is the most probable diagnosis?

100% Bubonic plague

0% Acute purulent lymphadenitis

0% Lymphogranulomatosis
0% Anthrax

0% Tularemia

The patient has developed pain in the axillary area, rise of temperature developed 10 hours ago.
On examination: shaky gait is marked, the tongue is coated by white coating. The pulse is
frequent. The painful lymphatic nodules are determined in the axillary area. The skin is
erythematous and glistering over the lymphatic nodules. What is the most probable diagnosis?

100% Bubonic plague

0% Acute purulent lymphadenitis

0% Lymphogranulomatosis

0% Anthrax

0% Tularemia

A 35-year-old patient has been suffering from an illness for 3 days. 5 days ago he returned from a
trip to Africa. The onset of disease was accompanied by fever up to 40oC, chills, acute headache,
myalgia. In the axillary region the lymph node enlarged up to 3x6 cm can be palpated. The lymph
node is dense, intensely painful, slightly mobile, without clear margins; the skin over the node is
hyperenic and tight. Tachycardia is present. Make the preliminary diagnosis:

100% Plague

0% Sepsis

0% Tularemia

0% Lymphadenitis

0% Anthrax

A 45-year-old patient, a sailor, was hospitalized on the 2nd day of the disease. A week ago he
returned from India. Complains of body temperature of 41oC, severe headache, dyspnea, cough
with frothy rusty sputum. Objectively: the patient is pale, mucous membranes are cyanotic,
breathing rate is 24/min, tachycardia is present. In lungs: diminished breath sounds, moist rales
over both lungs, crepitation. What is the most likely diagnosis?

100% Pneumonic plaque

0% Miliary tuberculosis
0% Influenza

0% Ornithosis

0% Sepsis

A 47-year-old woman, who 2 days ago returned from Peru, complains of pain and enlargement of
the lymph nodes in her right inguinal region. She was diagnosed with bubonic plague. What
medicine should be prescribed to the contact persons for urgent prevention of this disease?

Heterologous serum

Fluconazole

Human immunoglobulin

Doxycycline*

Chloroquine

ASPHYXIA
To replace the placeholder text on this page, you can just select it all and then start typing. But
don’t do that just yet!

MCQS

After birth a child was pale and had arrhythmical breathing. Oxygen therapy didn’t have any
effect. Pulse was weak and rapid. It was difficult to measure arterial pressure accurately. There
were no edemata. What is the most likely reason for these symptoms?

100% Asphyxia

0% Congestive heart failure


0% Intracranial haematoma

0% Intrauterine sepsis

0% Congenital pneumonia

TRACHEOESOPHAGEAL FISTULA
To replace the placeholder text on this page, you can just select it all and then start typing. But
don’t do that just yet!

MCQS

In a maternity hospital a newborn had been presenting with cough attacks after eating. The child
was discharged from the hospital on the 18th day due to a case of pneumonia. During the further
1,5 months the child had 2 cases of pneumonia. Periodically there are cough attacks after eating,
especially if the child lies on the left side. Objectively: the II degree hypotrophy, isolated moist
crackles, dyspnea. Stool and diuresis are not disrupted. What diagnosis is most likely?

100% Tracheoesophageal fistula

0% Mucoviscidosis

0% Posthypoxic encephalopathy

0% Hernia of the esophageal opening

0% Tracheobronchomalacia

A 65-year-old man complains of cough attacks that occur when he eats liquid foods. Three months
ago he was diagnosed with a carcinoma in the upper third of the esophagus. He underwent
radiation therapy. What complication developed in this man?

Perforation of a gastric cardia ulcer

Spontaneous pneumothorax
Tracheal stenosis

Lung abscess rupture into the pleural cavity

Tracheoesophageal fistula*

PLEURISY
To replace the placeholder text on this page, you can just select it all and then start typing. But
don’t do that just yet!

MCQS

A 32-year-old man complains of pain in the chest on the left, dyspnea, temperature rise up to
38,0°C, slight cough. The disease onset was 2 weeks ago after overexposure to cold. He had
suffered from bronchoadenitis in his childhood. The affected side lags during breating; percussion
reveals dull sound with oblique margin in the lower left lung, where breathing is absent. The right
heart border is displaced outwards. Mantoux test with 2 TU resulted in a papule 16 mm in size.
What diagnosis is most likely?

100% Tuberculous pleurisy

0% Central carcinoma of the left lung

0% Congestion pneumonia

0% Thromboembolism of the pulmonary artery branches

0% Community-acquired pneumonia

A 2-year-old child with persistent cough and subfebrile body temperature after a

case of URTI developed dyspnea, cyanosis of the nasolabial triangle, percussion

dullness and weakened respiration in the lower lobe of the right lung, and a slight

mediastinal displacement to the left. What pulmonary pathology is likely to cause this

clinical presentation?
Pleurisy*

Bronchitis

Pneumonia

Emphysema

Atelectasis

PNEUMOTHORAX
To replace the placeholder text on this page, you can just select it all and then start typing. But
don’t do that just yet!

MCQS

After playing with ’’mosaics’,’ a 2-year-old child suddenly developed a Cough, Stridorous
respirations, Urges to vomit, and Cyanosis against the background of relative somatic health.
What should the doctor suspect first when examining this child?

Foreign body aspiration*

Acute laryngotracheitis

Pneumonia

Acute obstructive bronchitis

Pertussis

A 26-year-old patient with left lower lobe pneumonia experiences an acute chest pain on the left
during coughing. Objectively: diffuse cyanosis, extension of the left side of chest. Percussion
reveals high tympanitis. Auscultation reveals no respiratory murmurs above the left side of chest.
There is a deviation of the right cardiac border towards the midclavicular line. What examination
will be the most informative?

100% X-Ray

0% Bronchoscopy

0% Bronchography

0% Pneumotachometry

0% Spirography

A 25 year old man has a sudden onset of chest pain on the right and dyspnea. His trachea is
deviated to the left. All of the following would be anticipated EXCEPT:

100% Pleural friction rub on the left

0% Absence of rhonchi

0% Absence of rales

0% Hyperresonance over the right chest

0% Distant breath sounds on the right

A patient complains about strong dyspnea that is getting worse during physical activity.
Presentations appeared suddenly 2 hours ago at work: acute chest pain on the left, cough. The
pain was abating, but dyspnea, dizziness, pallor, cold sweat and cyanosis were progressing.
Vesicular respiration is absent, X-ray picture shows a shadow on the left. What pathology might
be suspected?

100% Spontaneous left-sided pneumothorax

0% Pulmonary infarction

0% Pleuritis

0% Left-sided pneumonia

0% Pulmonary abscess
The patients has sustained blunt trauma to the chest. Which of the following would most likely be
the cause of acute cardiopulmonary collapse?

100% Pneumothorax

0% Hemothorax

0% Pulmonary contusion

0% Rib fractures

0% Acute adult respiratory distress syndrome (ARDS)

A patient complains about strong dyspnea that is getting worse during physical activity.
Presentations appeared suddenly 2 hours ago at work: acute chest pain on the left, cough. The
pain was abating, but dyspnea, dizziness, pallor, cold sweat and cyanosis were progressing.
Vesicular respiration is absent, X-ray picture shows a shadow on the left. What pathology might
be suspected?

100% Spontaneous left-sided pneumothorax

0% Pulmonary infarction

0% Pleuritis

0% Left-sided pneumonia

0% Pulmonary abscess

A 36 year old man was delivered to the surgical department an hour after a road accident. His
condition is getting worse: respiratory insufficiency is progressing, there are cardiac abnormalities.
Clinical and roentgenological investigations revealed mediastinal displacement. What process has
caused this complication?

100% Valvular pneumothorax

0% Open pneumothorax

0% Closed pneumothorax

0% Subcutaneous emphysema

0% Mediastinitis
A 38-year-old male patient complains of marked dyspnea that escalates with physical exertion.
The problems, namely acute chest pain on the left and cough, arose unexpectedly 2 hours before
at work. The pain abated, but there were progressing dyspnea, dizziness, pallor, cold sweat,
cyanosis. Auscultation reveals the absence of vesicular breath sounds, radiograph shows a shadow
on the left. What pathology can be suspected?

100% Left-sided spontaneous pneumothorax

0% Pulmonary infarction

0% Pleurisy

0% Left-sided pneumonia

0% Lung abscess

A 32-year-old woman was treated in an inpatient department for an abscess of the right lung. After
coughing, she suddenly developed labored breathing, cyanosis, and chest pain on the right. What
complication is most likely in this case?

Myocardial infarction

Esophageal perforation

Exudative pleurisy

Pyopneumothorax*

Infarction pneumonia

A 28-year-old man is undergoing a treatment for pulmonary tuberculosis. He has complains of


shortness of breath and a sharp chest pain on the right that appeared suddenly. Percussion detects
a bandbox resonance over the right lung, auscultation detects no breathing there. X-ray has shown
that the right lung is collapsed to its root by 1/2 of its volume, the heart and mediastinal organs
are shifted to the left. What complication has developed in this patient?

Dry pleurisy

Pleural empyema

Exudative pleurisy
Pulmonary infarction

Spontaneous pneumothorax*

A man suddenly developed a sharp pain in the right side of his chest. Dyspnea has rapidly
progressed. Objectively, the patient has marked acrocyanosis and is in a severe condition.
Subcutaneous emphysema is observed in the area of the patient’s neck and upper chest. Over the
right lung a bandbox resonance can be heard, respiration is absent there. The heart borders are
displaced to the left. The patient’s heart rate is 110/min., blood pressure -100/60 mm Hg. What is
the most likely disease in this case?

Myocardial infarction

Spontaneous pneumothorax*

Community-acquired pneumonia

Lung infarction

Exudative pleurisy

A 7-year-old boy after a fall from a height presents with rapid and shallow breathing and cyanotic
face. The right half of his thorax is distended and takes no part in the respiration. Percussion
detects tympanitis in the affected area, while auscultation detects no breathing there. What
pathology is the most likely cause of this clinical presentation? What instrumental examination
would be the most informative in this case?

Right-sided hemothorax. Survey X-ray of the chest

Mediastinitis. Survey X-ray of the chest

Right-sided tension pneumothorax. Chest X-ray*

Tension cyst of the right lung. Tracheobronchoscopy

Airway foreign body. Diagnostic and therapeutic bronchoscopy

A 40-year-old victim of a traffic accident sustained the following injuries: closed diaphyseal femur

fracture, brain concussion, multiple rib fractures, hemopneumothorax, degloving shin injuries.

What injuries require the most urgent attention?


Brain concussion

Closed diaphyseal femur fracture

Multiple rib fractures, hemopneumothorax*

All injuries are equivalent

Degloving shin injuries

A 45-year-old man diagnosed with acute pulmonary abscess suddenly developed

sharp pain in his chest on the right and dyspnea up to 30/min. Examination detects

facial cyanosis and shallow rapid respiration. Auscultation reveals acutely weakened

respiration throughout the whole right lung percussion reveals a vesicular tympanitic

(bandbox) resonance at the lung apex and dullness in the lower lobe. What

complication developed in this patient?

Esophageal perforation

Pyopneumothorax*

Acute mediastinitis

Pneumothorax

Pleuropneumonia

A 32 y.o. patient who has been staying in a hospital on account of acute abscess of his right lung
suddenly felt pain after coughing in the right half of thorax, he got heavy breathing, cyanosis.
What complication is the most probable?

100% Pyopneumothorax

0% Infarction-pneumonia
0% Myocardial infarction

0% Esophagus perforation

0% Exudative pleurisy

A 64-year-old patient complains of severe pain in the right side of chest, dyspnea, dry cough
which appeared suddenly on exertion. Objectively: the right side of the chest lags behind in the
act of breathing. Percussion reveals tympanic sound. Auscultation reveals pronouncedly
diminished

breath sounds on the right. Ps-100/min, weak, arrhythmic. AP- 100/50 mm Hg. Cardiac sounds are
decreased. What disease can be suspected in this patient?

100% Right-sided pneumothorax

0% Right-sided hydrothorax

0% Right-sided dry pleurisy

0% Right-sided pleuropneumonia

0% PATE

A child undergoes in-patient treatment for acute staphylococcal destruction of the right lung.
Unexpectedly he develped acute chest pain on the right, dyspnea, cyanosis. The right side of chest
lags behind in the respiratory act. Percussion reveals dullness in the lower parts on the right,
bandbox resonance in the upper parts. Borders of the relative cardiac dullness are shifted to the
left. What complication has most likely developed?

100% Right-sided pyopneumothorax

0% Pleural empyema

0% Spontaneous pneumothorax

0% Exudative pleuritis

0% Right lung abscess


A 36-year-old man was delivered to the surgical department an hour after a road accident. His
condition is getting worse: respiratory insufficiency is progressing, there are cardiac abnormalities.
Clinical and roentgenological investigations revealed mediastinal displacement. What process has
caused this complication?

100% Valvular pneumothorax

0% Open pneumothorax

0% Closed pneumothorax

0% Subcutaneous emphysema

0% Mediastinitis

A hospital has admitted a 52-year-old patient with disseminated pulmonary tuberculosis,


complaints of acute pain in the right half of chest, that developed after going upstairs to the 3rd
floor; cough, dyspnea at rest, marked cyanosis. What kind of complication should suspected first
of all?

100% Spontaneous pneumothorax

0% Cardiac failure

0% Pulmonary failure

0% Pleuritis

0% Acute myocardial infarction


A 25 y.o. patient was admitted with chest trauma. Clinical and X-ray examination have revealed
tense pneumothorax on the left. What emergency treatment should be undertaken?

100% Pleural cavity drainage

0% Intravenous infusions

0% Oxigenotherapy

0% Intubation

0% Analgetics

Two days ago a woman fell from the height of 1,5 m. She complains of severe thoracic pain on the
left and dyspnea. Chest X-ray reveals hydropneumothorax on the left with fluid level at the 7th rib
and the lung collapsed by 1/3. The 6th-7th ribs are fractured along the scapular line.
Serohemorrhagic fluid was obtained during thoracic puncture. What treatment tactics should be
chosen?

100% Thoracocentesis on the left at the 7th intercostal space

0% Thoracocentesis at the 2nd intercostal space

0% Repeated puncture of the pleural cavity

0% Intubation of trachea with artificial pulmonary ventilation

0% Alcohol-novocaine block of the fracture area

A patient was delivered to a surgical department after a road accident with a closed trauma of
chest and right-sided rib fracture. The patient was diagnosed with right-sided pneumothorax; it is
indicated to perform drainage of pleural cavity. Pleural puncture should be made:

100% In the 2nd intercostal space along the middle clavicular line

0% In the 6th intercostal space along the posterior axillary line

0% In the 7th intercostal space along the scapular line

0% In the projection of pleural sinus

0% In the point of the greatest dullness on percussion

A 29-year-old unconscious patient has been delivered to a hospital. Objectively: skin and mucous
membranes are pale, cyanotic, breath sounds are dramatically diminished on the right and cannot
be auscultated in the lower parts, at the level of the 6 rib along the anterior axillary line there is a
wound hole with moderate bleeding and passage of air during inspiration. Radiography reveals a
bullet in the pleural cavity. What is the medical tactics of choice?

100% Emergency thoracotomy

0% Thoracoscopy with removal of bullet

0% Converting a tension pneumothorax into a simple (open) pneumothorax

0% Thoracostomy

0% Tight bandage on a wound

Examination of a 38-year-old patient who had been hit with a blunt object on the left side of chest
revealed a fracture of the X rib with fragments displacement, parietal pneumothorax. The patient
complains of pain in the left subcostal area. Objectively: the patient is pale, AP- 80/40 mm Hg, Ps-
138/min, of poor volume. USI reveals fluid in the left abdomen. Splenic rupture is present. What
treatment tactics should be chosen?

100% Drainage of the left pleural cavity followed by laparotomy

0% Immediate upper median laparotomy followed by drainage of the left pleural cavity

0% Immediate laparotomy and alcohol-novocaine block of the X rib

0% Anti-schock actions followed by laparotomy after the arterial pressure rise

0% Left-sided thoracotomy immediately followed by laparotomy

Examination of a 38-year-old patient who had been hit with a blunt object on the left side of chest
revealed a fracture of the X rib with fragments displacement, parietal pneumothorax. The patient
complains of pain in the left subcostal area. Objectively: the patient is pale, AP- 80/40 mm Hg, Ps-
138/min, of poor volume. USI reveals fluid in the left abdomen. Splenic rupture is present. What
treatment tactics should be chosen?

100% Drainage of the left pleural cavity and laparotomy

0% Immediate upper middle laparotomy and following drainage of the left pleural cavity

0% Immediate laparotomy and alcohol-novocaine block of the X rib

0% Anti-schock actions and laparotomy following the arterial pressure rise

0% Left-sided thoracotomy and immediately following laparotomy

On the 4th day after suturing the perforative stomach ulcer a patient with pulmonary emphysema
developed spontaneous pneumothorax. What is the best place for pleural drainage?
100% The second intercostal space along the medioclavicular line

0% The eighth intercostal space along the posterior axillary furrow

0% The seventh intercostal space along the anterior axillary furrow

0% The sixth intercostal space along the anterior axillary furrow

0% The fifth intercostal space along the medioclavicular line

A 36 y.o. patient is diagnosed with right sided pneumothorax. What method of treatment is
indicated to the patient?

100% Surgical drainage of the pleural cavity

0% Antiinflammation therapy

0% Symptomatic therapy

0% Pleural puncture

0% Thoracotomy

A 35-year-old woman had acute onset of the disease that started with fever up to 39.0°C and
cough. 3 days later her dyspnea at rest increased up to 35/min. Downward from her right
shoulder-blade angle, percussion detects a dull sound. No vocal fremitus, respiratory sounds
cannot be auscultated. What is the treatment tactics?

Artificial lung ventilation

Oxygen therapy

Antibiotic therapy

Physiotherapy

Pleural tap*

After lifting a heavy object, a 41-yearold man felt a sharp pain in his chest on the right and
developed progressing shortness of breath. The man’s condition is severe. His lips and mucosa are
cyanotic, respiratory rate – 28/min., pulse – 122/min. Over the right half of the chest, percussion
shows tympanitis, while auscultation detects sharply weakened breathing. The II heart sound is
accentuated over the pulmonary artery. His blood pressure is 80/40 mm Hg. What primary
emergency measure must be taken at the prehospital stage in this case?

Administration of adrenaline

Administration of euphyllin (aminophylline)

Oxygen inhalation

Aspiration of air from the pleural cavity*

Call for a cardiology team

A 40-year-old man was brought into a hospital with a closed chest trauma. Examination shows
that the right side of his chest lags behind during breathing. Palpation detects a sharp pain in the
projection of ribs V, VI, and VII on the anterior axillary line and subcutaneous emphysema on the
right side of the torso. Percussion reveals tympanitis over the right-sided segments of the chest.
Auscultation detects no breathing on the right and vesicular breathing on the left. What surgical
procedure is necessary for this patient?

Drainage of subcutaneous emphysema

Drainage of the right pleural cavity*

Immediate thoracotomy

Splint stabilization of the rib fracture

Tight bandaging of the chest

A 62-year-old patient has been hospitalized with complaints of pain in the thorax

on the right during breathing, dyspnea, and dry cough. Ten days ago he slipped and

fell hitting his right side. On examination: the patient lies on the left side. The right

side of the thorax lags during breathing. On the right there is crepitation and pain in

the III-IV ribs. Dullness of percussion sound and sharply diminished breath sounds

can be observed. On X-ray: signs of exudate, fracture of the IIIIV ribs. On

pleurocentesis: blood is detected. Choose the further tactics.


Transfer to a thoracic surgery department*

Apply a fixation bandage to the rib cage

Refer to a traumatologist

Perform repeated pleural taps

Prescribe conservative therapy

4 weeks after myocardial infarction a 56-year-old patient developed acute heart pain, marked
dyspnea. Objectively: the patient’s condition is extremely grave, there is marked cyanosis of face,
swelling and throbbing of neck veins, peripheral pulse is absent, the carotid artery pulse is
rhythmic, 130/min., BP is 60/20 mm Hg. Auscultation of the heart reveals extremely muffled
sounds, percussion reveals heart border extension in both directions. What is the optimal
treatment tactics for this patient?

100% Pericardiocentesis and immediate thoracotomy

0% Oxygen inhalation

0% Puncture of the pleural cavity on the left

0% Conservative treatment, infusion of adrenomimetics

0% Pleural cavity drainage

A 24-year-old patient had been delivered to a thoracic department with a chest injury, fracture of
the IV, V VI ribs on the right. Plan radiography showed the fluid level in the pleural cavity to be
reaching the III rib on the right. Puncture contained blood clots. What is the optimal treatment
tactics?

100% Emergency thoracotomy

0% Pleural puncture

0% Thoracentesis and thoracostomy

0% Hemostatic therapy

0% Medical thoracoscopy
A 29-year-old female patient complains of dyspnea, heaviness and chest pain on the right, body
temperature rise up to 37,2oC. The disease is associated with a chest trauma received 4 days ago.

Objectively: skin is pale and moist. Ps- 90/min., regular. Palpation reveals a dull sound on the
right, auscultation reveals significantly weakened vesicular breathing. In blood: RbCs- 2,8 • 1012/l,
colour index -0,9, Hb- 100 g/l, WBCs- 8,0 • 109/l, ESR- 17 mm/hour. What results of diagnostic
puncture of the pleural cavity can be expected?

100% Haemorrhagic punctate

0% Chylous liquid

0% Exudate

0% Transudate

0% Purulent punctate

A 45-year-old male patient with acute abscess of the left lung has suddenly developed acute chest
pain and dyspnea while coughing, tachycardia has increased. The control Ro-gram shows left lung
collapse, the air in the left pleural cavity and a horizontal fluid level. What is the mechanism of
this complication?

100% Abscess burst into the pleural cavity

0% Bullae rupture of the left lung

0% Inflammation spread to the visceral pleura

0% Atelectasis of the left lung

0% Acute cardiovascular insufficiency

A 35 y.o. patient was admitted to the local hospital a week after a road accident with clinical
picture of clotted hemothorax. What is the most appropriate treatment tactic for prevention of
acute pleural empyema?

100% Surgical removal of clotted hemothorax

0% Treatment by pleural punctions

0% Complex conservative therapy

0% Passive drainage of pleural cavity


0% Active drainage of pleural cavity

A 41 y.o. patient complains of acute pain in the right side of the thorax and sudden progress of
dyspnea following the lifting of a heavy object. The patient’s condition is grave: lips and mucous
membranes are cyanotic, BR- 28/min, Ps-122 bpm., AP- 80/40 mm Hg. There is tympanitis on
percussion and weakened breathing on auscultaion on the right. S2 sound is accentuated above
pulmonary artery. What is the main urgent measure on the prehospital stage?

100% Air aspiration from the pleural cavity

0% Epinephrine introduction

0% Euphilline introduction

0% Call for cardiologic team

0% Oxygen inhalation

A 24 y.o. male patient was transferred to the chest surgery department from general surgical
department with acute post-traumatic empyema of pleura. On the X-ray: wide level horizontal of
fluid on the right. What method of treatment should be prescribed?

100% Punction and drainage of pleural cavity

0% Decortication of pleura

0% Pneumoectomy

0% Thoracoplasty

0% Lobectomy

A 42 y.o. patient lifted a heavy object that resulted in acute pain in the right half of his chest,
increased dyspnea. The patient’s condition is grave: cyanosis of lips and mucous membranes, RR is
28 pm, Ps-122 bpm. On percussion there is tympanitis above the right half of chest, on
auscultation - stongly diminished breath sounds; accent of the II heart sound above the
pulmonary artery. AP is 80/40 mm Hg. What is the main emergency action at the pre-admission
stage?

100% Air aspiration from the pleural cavity

0% Adrenaline introduction
0% Aminophylline introduction

0% Calling a cardiological brigade

0% Oxygen inhalation

A 21-year-old female patient has been hospitalized on an emergency basis because of severe
dyspnea, pain in the left side of chest. Body temperature is 38,8°C. The condition developed three
days ago. Respiratory rate is 42/min, auscultation reveals shallow breathing. There is percussive
dullness on the right starting from the middle of the blade, breath sounds cannot be heard. The
left border of heart is 3 cm displaced outwards. Embryocardia is present, HR is 110/min. The right
hypochondrium is painful on palpation. What urgent therapeutic measures should be taken in
this situation?

100% Emergency puncture of the pleural cavity

0% Administration of penicillin antibiotics

0% Injection of Lasix

0% Injection of cardiac glycosides

0% Transferring the patient to the thoracic surgery department

A 35-year-old victim of a road accident has got an injury of the right side of his chest. Objectively:
respiration rate - 28-30/min, respiration is shallow, restricted respiratory excursion and
acrocyanosis are present. Ps-110 bpm, AP- 90/60 mm Hg. Respiratory sounds over the right lung
cannot be auscultated. Chest radiograph shows fractures of the VI-VII ribs on the right, the right
pleural cavity contains both air and fluid, with the fluid at about the level of the V rib, the shadow
of the mediastinum is displaced to the left. What first aid should be provided to the victim?

100% Puncture of the pleural cavity

0% Antibiotic administration

0% Vagosympathetic blockade

0% Artificial ventilation of lungs

0% Urgent thoracotomy

A patient undergoing treatment for the left-sided destructive pneumonia presents with
deterioration of his general condition, progressing dyspnea, cyanosis. Objectively: the left side of
chest is not involved in respiration, breathing sounds cannot be auscultated. Radiograph shows a
shadow reaching the 5 rib with a horizontal fluid level and a radiolucency above it, the
mediastinum is displaced to the right. What is the medical tactics?
100% Thoracostomy

0% Open thoracotomy

0% Endotracheal intubation

0% Infusion and antibacterial therapy

0% Emergency bronchoscopy

A 19-year-old student has been hospitalized on an emergency basis because of severe dyspnea,
pain in the left side of chest. The patient got sick 3 days ago. Objectively: body temperature
-38,8oC. Bh - 42/min., hypopnoe. There is dullness of percussion sound on the right of the scapula
middle, breathing sounds cannot be auscultated. The left border of heart is displaced outwards by
3 cm. Embryocardia is preent, HR -110/min. The right hypochondrium is painful on palpation.
What are the immediate treatment measures in this situation?

100% Urgent puncture of pleural cavity

0% Administartion of antibiotics of penicillin group

0% Injection of lasix

0% Injection of cardiac glycosides

0% Transfer of the patient to the thoracic surgery department

A 38-year-old patient has suddenly developed pain in the left side of his chest, suffocation.
Objectively: moderately grave condition, Ps- 100/min, AP- 90/60 mm Hg, breath sounds on the
left cannot be auscultated. Chest radiography shows the collapse of the left lung up to 1/2. What
kind of

treatment should be administered?

100% Passive thoracostomy

0% Rest, resolution therapy

0% Pleural puncture

0% Operative therapy

0% Active thoracostomy
A 24-year-old patient had been delivered to the thoracic department with a chest injury, a fracture
of the IV, V VI ribs on the right. Plan radiography shows the fluid level in the pleural cavity
reaching the III rib on the right. Puncture blood contained clots. What is the optimal treatment
tactics?

100% Emergency thoracotomy

0% Pleural puncture

0% Thoracentesis and thoracostomy

0% Hemostatic therapy

0% Medical thoracoscopy

PLEURAL EMPYEMA
To replace the placeholder text on this page, you can just select it all and then start typing. But
don’t do that just yet!

MCQS

A 52-year-old man had an acute onset of the disease after an overexposure to cold that occurred 3
weeks ago. He complains of cough, high body temperature of 30.5°C, chest pain on the right, and
marked dyspnea. His pulse is 120/min., blood pressure – 90/60 mm Hg, respiration rate – 48/min.
Acrocyanosis is observed. Objectively, the right side of his chest lags behind during breathing.
Percussion detects a pulmonary dullness on the right. Auscultation detects no breathing over the
right lung. X-ray shows on the right a shadow with fuzzy upper margin to the level of rib II.
Exudate contains 90% of neutrophils. What is the most likely diagnosis in this case?

Infiltrative tuberculosis of the right lung

Exudative tuberculous pleurisy

Lung cancer

Pleural empyema*
Croupous pneumonia

A patient with bilateral hydrothorax has repeatedly undergone pleural puncture on both sides.
After a regular puncture the patient’s condition has become worse: he presents with fever, chest
pain. The next day, the attending physician performing pleural puncture revealed some pus on the
right. What is the mechanism of acute right-sided empyema development?

100% Contact-and-aspiration

0% Lymphogenous

0% Hematogenous

0% Implantation

0% Aerial

MEDIASTINITIS
To replace the placeholder text on this page, you can just select it all and then start typing. But
don’t do that just yet!

MCQS

A 36 y.o. patient was admitted to the hospital with sharp pain in substernal area following
occasional swallowing of a fish bone. On esophagoscopy the foreign body wasn’t revealed. The
pain increased and localized between scapulas. In a day temperature elevated, condition became
worse, dysphagia intensified. What complication has developed?

100% Perforation of esophagus with mediasti-nitis

0% Esophageal hemorrhage

0% Obstruction of esophagus

0% Pulmonary atelectasis
0% Aspirative pneumonia

A 42 y.o. patient was admitted 3 hours after a trauma with evident subcutaneous emphysema of
the upper part of his body, dyspnea, tachycardia 120/min. X-ray examination revealed no
pneumothorax, significant dilatation of mediastinum to the both sides. What emergency care is
needed?

100% Drainage of anterior mediastinum

0% Pleural cavity punction

0% Pleural cavity drainage

0% Toracoscopy

0% Toracotomy

After the pneumatic dilatation of oesophageal stricture a patient developed acute retrosternal pain
getting worse when throwing the head back and swallowing. Objectively: dilatation of the neck
veins, dropped beat pulse, signs of purulent intoxication, oliguria, emphysema of the upper
portion of chest. What disease can be suspected?

100% Suppurative mediastinitis

0% Thrombosis of the superior vena cava

0% Pleural empyema

0% Acute myocardial infarction

0% Spontaneous pneumothorax

Esophagus wall of a 72 year old patient with severe concomitant pathology was injured during
urgent fi-broesophagogastroscopy. This resulted in progressing of acute respiratory failure and
collapse of the left lung. What aid should be rendered?

100% Drainage of pleural cavity by Bullaux method, mediastinum drainage, antibacterial

0% Left-sided thoracotomy, closure of esophagus wound 0% Endoscopic closure of


esophagus wound, drainage
PLEURITIS
To replace the placeholder text on this page, you can just select it all and then start typing. But
don’t do that just yet!

MCQS

A 52 y.o. male patient has become ill gradually. There is pain in the left side of the thorax during 2
weeks, elevation of temperature till 38 - 39° C. On examination: left chest side falls behind in
breathing movement no voice tremor over the left lung. Dullness that is more intensive in lower
parts of this lung. Right heart border is deviated outside. Sharply weakened breathing over the left
lung, no rales. Heart sounds are mufflet, tachycardia. What is the most probable diagnosis?

100% Exudative pleuritis

0% Spotaneous pneumothorax

0% Atelectasis of lung

0% Cirrhotic tuberculosis

0% Infarction-pneumonia

A 26 year old man was admitted to the hospital because of stabbing back pain on inspiration and
dyspnea. Examination results: BT of 37°C, Ps of 24/min, HR of 92/min, vesicular breath sounds.
There is a dry, grating, low-pitched sound heard on both expiration and inspiration in the left
inferior lateral part of the chest. What is the most likely diagnosis?

100% Acute fibrinous pleuritis

0% Myocarditis

0% Pneumonia

0% Acute bronchitis

0% Pneumothorax

A 26-year-old male patient complains of piercing pain during breathing, cough, dyspnea.
Objectively: to- 37,3oC, respiration rate is 19/min, heart rate is 92/min; BP is 120/80 mm Hg.
Vesicular respiration is observed. In the inferolateral parts of chest auscultation in both
inspiration and expiration phase revealed noise that was getting stronger at phonendoscope
pressing and could be still heard after cough. ECG showed no pathological changes. What is the
most likely diagnosis?
100% Acute pleuritis

0% Intercostal neuralgia

0% Subcutaneous emphysema

0% Spontaneous pneumothorax

0% Pericarditis sicca

PULMONARY INFARCTION
To replace the placeholder text on this page, you can just select it all and then start typing. But
don’t do that just yet!

MCQS

On the 4th day after a surgery for a cystoma in the right ovary, a woman suddenly developed a
fever of 37.7oC and a chest pain on the right with expectoration of pink sputum. Examination of
the lungs has revealed a dull pulmonary sound and isolated wet crackles in the lower right
segments. What complication is most likely developed in this patient?

Pneumothorax

Pulmonary infarction*

Pneumonia

Pulmonary abscess

Exudative pleurisy

A woman has developed sudden thoracic pain on the right with expectoration of pink sputum and
body temperature rise up to 37, 7oC on the 4th day after the surgery for cystoma of the right ovary.
On lung examination: dullness of the lung sound on the lower right is observed. Isolated moist
crackles can be auscultated in the same area. What complication is the most likely?
100% Pulmonary infarction

0% Pneumonia

0% Pulmonary abscess

0% Exudative pleurisy

0% Pneumothorax

PULMONARY ABSCESS
To replace the placeholder text on this page, you can just select it all and then start typing. But
don’t do that just yet!

MCQS

A 35 y.o. woman was admitted to thoracic surgery department with fever up to 40° C, onset of pain
in the side caused by deep breathing, cough with considerable quantity of purulent sputum and
blood with bad smell. What is the most likely diagnosis?

100% Abscess of the lung

0% Complication of liver echinococcosis

0% Bronchiectatic disease

0% Actinomycosis of lungs

0% Pulmonary tuberculosis

A 32-year-old patient has developed an acute condition after hypothermia: temperature - 40oC,
cough with 200 ml of sputum per day. The sputum is purulent, foul-smelling. To the right of the
lower lobe the mixed moist rales can be auscultated. Blood tst results: WBCs -18,0 • 109/l, ESR - 45
mm/h. Radiographically: in the lower lobe of the right lung there is a thick-walled cavity up to 6
cm in diameter with a high horizontal level. What is the most likely diagnosis?

100% Lung abscess

0% Fibro-cavernous pulmonary tuberculosis


0% Lung cyst

0% Decomposing lung carcinoma

0% Infiltrative pulmonary tuberculosis

X-ray pattern of thorax organs revealed a large intensive inhomogeneous opacity with indistinct
outlines on the right side at the level of the 4-th rib. In the centre of this opacity there is a
horizontal level and clearing of lung tissue above it. What disease does this X-ray pattern
correspond with?

100% Abscess of the right lung

0% Peripheral cancer

0% Tuberculoma of the right lung

0% Right-sided pneumothorax

On the 4th day after recovery from a cold a patient was hospitalized with complaints of solitary
spittings of mucoid sputum. On the 2nd day there was a single discharge of about 250 ml of
purulent blood-streaked sputum. Objectively: the patient’s condition is moderately severe.
Respiratory rate

-28-30/min., Ps- 96/min., BP- 110/70 mm Hg. Respiration over the left lung is vesicular, over the
right lung - weakened. There are various moist crackles over the lower lobe and amphoric breath
sounds near the angle of scapula. What is the most likely diagnosis?

100% Acute pulmonary abscess

0% Exudative pleuritis

0% Acute focal pneumonia

0% Pleural empyema

0% Pyopneumothorax

The patient’s respirations are 28/min., tachycardia is up to 100/min., there are signs of
intoxication. In the area of the right scapula the percussion sound is dull, and the respiration is
bronchial, with single fine vesicular crackles and crepitation sounds. X-ray shows a massive
inflammatory infiltration in the middle pulmonary field. Three days later, against the background
of treatment, the patient developed a cough attack, during which he expectorated 200 ml of
purulent sputum. After that, his body temperature dropped to sub-febrile values and his overall
condition improved. At the level of the scapular angle, against the background of the pulmonary
infiltration, there is a round area of radiolucency with the horizontal level of liquid. Make the
diagnosis:

Pulmonary abscess*

Multiple bronchiectases

Pleural empyema

Right-sided bronchitis

Gangrene of the right lung

For the last 2 months, a 29-year-old woman has been complaining of chest pain on the left, cough,
shortness of breath, and fever of 39.6oC. The left half of her chest lags behind in the act of
breathing, her vesicular respiration is weakened, and there is a shortening of the percussion sound
on the left. X-ray has shown a round shadow in the lower lobe of the left lung.

Make the right diagnosis:

Pleural empyema

Lung cancer

Purulent pleurisy

Chronic pneumonia

Lung abscess*

A 42-year-old male patient has been delivered to a hospital in a grave condition with dyspnea,
cough with expectoration of purulent sputum, fever up to 39, 5oC. The first symptoms appeared 3
weeks ago. Two weeks ago, a local therapist diagnosed him with acute right-sided pneumonia.
Over the last 3 days, the patient’s condition deteriorated: there was a progress of dyspnea,
weakness, lack of appetite. Chest radiography confirms a rounded shadow in the lower lobe of the
right lung with a horizontal fluid level, the right sinus is not clearly visualized. What is the most
likely diagnosis?

100% Abscess of the right lung

0% Acute pleuropneumonia
0% Right pulmonary empyema

0% Atelectasis of the right lung

0% Pleural effusion

A 45-year-old patient with acute abscess of the left lung has suddenly developed acute chest pain
and dyspnea while coughing, tachycardia has increased. The control Ro-gram shows collapsed left
lung, the air in the left pleural cavity and a horizontal fluid level. What is the mechanism of this
complication?

100% Abscess burst into the pleural cavity 0% Bullae rupture of the left lung

0% Inflammation spread to the visceral pleura

0% Atelectasis of the left lung

0% Acute cardiovascular insufficiency

RESPIRATORY DISTRESS
To replace the placeholder text on this page, you can just select it all and then start typing. But
don’t do that just yet!

MCQS

A patient has undergone an operation on account of perforated ulcer of stomach, terminal phase
of diffuse peritonitis and endotoxic shock. In the postoperative period he is prescribed artificial
pulmonary ventilation with 60% oxygen inhalation. Blood gases: Pa02- 7078 mm Hg, hypoxemy
doesn’t ecrease, CVP (central venous pressure) - 150180 mm of water column, AP- 90/60 mm Hg
(against the backgound of taking big doses of dopamine). Radiogram shows diffuse pulmonary
infiltration. What cause the refractory arterial hypoxemia?

100% Respiratory distress syndrome


0% Bilateral pneumonia

0% Pneumothorax

0% Mendelson’s syndrome

0% Pulmonary edema

ATELECTASIS
To replace the placeholder text on this page, you can just select it all and then start typing. But
don’t do that just yet!

MCQS

A 65-year-old man complains of dyspnea, severe cough with expectoration of

small amounts of blood-streaked sputum, weight loss, body temperature 37.2°C. loss

of appetite, and weakness. He has been suffering from this condition for years. The

patient’s condition deteriorated one year ago, dyspnea developed 3 weeks ago. The

patient is a lifelong smoker. He is a carpenter by occupation. Objectively he is of

normal body type but emaciated. Right side of the chest is retracted, excursions are

limited, accessory muscles take part in the respiration, respiratory rate is 22 /min

percusion detects dull sound over the upper segment. Chest X-ray shows shrunken

right upper lobe with homogenous shadow connected to the root of the lung: the root

is deformed: mediastinal organs are displaced to the right. What is the most likely

diagnosis?

Fibrosing alveolitis
Complete right-sided pneumothorax

Pulmonary sarcoidosis

Obstructive atelectasis*

Pulmonary tuberculosis

A 4 year old girl was playing with her toys and suddenly she got an attack of cough, dyspnea.
Objectively: respiration rate - 45/min, heart rate - 130/min. Percussion revealed dullness of
percutory sound on the right in the lower parts. Auscultation revealed diminished breath sounds
with bronchial resonance on the right. X-ray pictue showed shadowing of the lower part of lungs
on the right. Blood analysis revealed no signs of inflammation. The child was diagnosed with
foreign body in the right bronchus. What complication caused such clinical presentations?

100% Atelectasis

0% Emphysema

0% Pneumothorax

0% Bronchitis

0% Pneumonia

LARYNGEAL EDEMA
To replace the placeholder text on this page, you can just select it all and then start typing. But
don’t do that just yet!

MCQS

After the contact with chemicals a plant worker has suddenly developed stridor, voice hoarseness,
barking cough, progressing dyspnea. Objective examination reveals acrocyanosis. What is the
provisional diagnosis?

100% Laryngeal edema


0% Laryngeal carcinoma

0% PATE

0% Pulmonary atelectasis

0% Pneumothorax

PLEURAL EFFUSION
To replace the placeholder text on this page, you can just select it all and then start typing. But
don’t do that just yet!

MCQS

Examination of a 22 year old man suffering from polyarthralgia and high fever revealed right-
sided exudative pleuritis. X-ray picture showed a homogenous shadow below the IV rib on the
right. In the II segment there were single dense focal shadows. Mantoux test with 2 TU resulted in
formation of a papula 16 mm large. Pleural liquid has increased protein concentration, Ri-valta’s
reaction is positive, there was also increased number of leukocytes with prevailing lymphocytes.
What is the most probable etiology of pleuritis?

100% Tuberculous

0% Cancerous

0% Staphylococcal

0% Viral

0% Autoimmune

A 57-year-old patient complains of dyspnea at rest. The patient presents with orthopnea,
acrocyanosis, bulging cervical veins. On percussion: dull sound over the lower lung segments; on
auscultation: no respiratory murmurs. Heart rate is 92/min. Right-sided cardiac dilatation is
observed. The liver is enlarged by 7 cm. Shins are swollen. Pleural effusion is suspected. What
indicator would confirm the presence of transudate in this case?

100% Total protein content in the pleural fluid below 25 g/l


0% Presence of atypical cells

0% Total protein content in the pleural fluid above 30 g/l

0% Specific gravity exceeding 1015

0% Positive Rivalta’s test

PULMONARY FIBROSIS
To replace the placeholder text on this page, you can just select it all and then start typing. But
don’t do that just yet!

MCQS

A 49-year-old patient 2 years ago was diagnosed with stage 1 silicosis. He complains of increased
dyspnea and pain in the infrascapular regions. X-ray shows diffuse intensification and distortion of
lung pattern with numerous nodular shadows 2-4 mm in diameter. Hardening of right interlobar
pleura is detected. Lung roots are hardened and expanded. What X-ray type of pneumosclerosis
does the patient have?

100% Nodular

0% Interstitial

0% Interstitial-nodular

0% Nodal

0% Tumor-like

A 27-year-old woman has been treated in a surgical department for pleural empyema for 6
months. Multiple paracenteses of the pleural cavity were performed along with antibacterial
treatment. The patient’s condition is slowly aggravating; attempts to fully stretch the lung were
unsuccessful. Choose the tactics:

100% Decortication of the lung

0% Change antibiotics

0% Set constant active suction drain

0% Pulmonectomy

0% Include hyperbaric oxygenation in the treatment

Two years ago, a 46-year-old patient was diagnosed with stage I silicosis. Currently the patient
complains of escalating dyspnea, pain in the infrascapular regions. Radiograph shows a diffuse
enhancement and distortion of lung markings, as well as multiple nodular shadows 2-4 mm in
diameter. There is interlobar pleural density on the right. Dense shadows are found in the hilar
regions. Specify the form of radiographic pulmonary fibrosis in this case:

100% Nodular

0% Interstitial

0% Interstitial nodular

0% Nodal

0% Tumor-like

PULMONARY EDEMA
To replace the placeholder text on this page, you can just select it all and then start typing. But
don’t do that just yet!

MCQS

A 65-year-old man with acute anterior myocardial infarction developed an asphyxia attack.
Examination detected diffuse cyanosis. In the lungs there are numerous heterogenous wet
crackles. Heart rate is 100/min. Blood pressure is 120/100 mm Hg. What complication occurred in
this patient?
Ventricular septal rupture

Cardiogenic shock

Hypertensive crisis

Pulmonary edema*

Pulmonary embolism

REFLEX
To replace the placeholder text on this page, you can just select it all and then start typing. But
don’t do that just yet!

MCQS

A 49-year-old male patient complains of dyspnea of combined nature, cough, shin edemata,
abdomen enlargement due to ascites. He has a 20-year history of chronic bronchitis. For the last 3
years he has been disabled (group II) because of cardiac changes. Objectively: mixed cyanosis,
edemata. Ps - 92/min, rhythmic, AP - 120/70 mm Hg, respiration rate - 24/min. There is
accentuation of the second sound above the pulmonary artery. Auscultation reveals the box
resonance above the lungs. There are also dry rales over the entire surface of lungs. What is the
mechanism of heart changes development in this patient?

100% Euler-Liljestrand reflex

0% Kitaev’s reflex

0% Bainbridge reflex

0% Cardiovascular reflex

0% Respiratory reflex
RESPIRATORY FAILURE
To replace the placeholder text on this page, you can just select it all and then start typing. But
don’t do that just yet!

MCQS

A 3 month old infant suffering from acute segmental pneumonia has dyspnea (respiration rate - 80
per minute), paradoxical breathing, tachycardia, total cyanosis. Respiration and pulse - ratio is 1:2.
The heart dullness under normal size. Such signs characterise:

100% Respiratory failure of III degree

0% Respiratory failure of I degree

0% Respiratory failure of II degree

0% Myocarditis

0% Congenital heart malformation

Physiology of pregnancy
To replace the placeholder text on this page, you can just select it all and then start typing. But
don’t do that just yet!

MCQS

A famil
Physiology of pregnancy
To replace the placeholder text on this page, you can just select it all and then start typing. But
don’t do that just yet!

MCQS

A famil

DERMATITIS
To replace the placeholder text on this page, you can just select it all and then start typing. But
don’t do that just yet!

MCQS

A child was taken to a hospital with focal changes in the skin folds. The child was anxious during
examination, examination revealed dry skin with solitary papulous elements and ill-defined
lichenification zones. Skin eruption was accompanied by strong itch. The child usually feels better
in

summer, his condition is getting worse in winter. The child has been on bottle feeding since he
was 2 months old. He has a history of exudative diathesis. His grandmother on his mother’s side
has bronchial asthma. What is the most likely diagnosis?

100% Atopic dermatitis

0% Contact dermatitis

0% Seborrheal eczema

0% Strophulus

0% Urticaria
Physiology of pregnancy
To replace the placeholder text on this page, you can just select it all and then start typing. But
don’t do that just yet!

MCQS

A famil

OPHTHALMOLOGY
To replace the placeholder text on this page, you can just select it all and then start typing. But
don’t do that just yet!

MCQS

A 10-year-old boy complains of pain in his left eye and strong photophobia after he has injured his
left eye with a pencil at school. Left eye examination revealed: blepharospasm, ciliary and
conjunctival congestion, cornea is transparent, other parts of eyeball have no changes. Visus 0,9.
Right eye is healthy, Visus 1,0. What additional method would you choose first of all?

100% Staining test with 1% fluorescein

0% X-ray examination of orbit

0% Tonometria

0% Gonioscopia

0% Cornea sensation-test

PNEUMOCISTA
To replace the placeholder text on this page, you can just select it all and then start typing. But
don’t do that just yet!
MCQS

A 27-year-old man complains of dry cough, dyspnea during the slightest exertion, chest pain, and
high temperature of 37.3°C that persists for the last 3 weeks. He has a past history of drug abuse.
His respiration is rough, without crackles. He has tachycardia of 120/min. X-ray shows interstitial
changes on the both sides of his lungs. Bronchoscopy detects Pneumocista carinii in the lavage
fluid. What medicine will be the most effective for the treatment of this patient?

Biseptol (co-trimoxazole), clindamycin*

Interferon, rimantadine

Ampicillin, nizoral (ketoconazole)

Erythromycin, rifampicin

Cefamezin (cefazolin), nitroxoline

CANCER OF LUNG
To replace the placeholder text on this page, you can just select it all and then start typing. But
don’t do that just yet!

MCQS

A 72-year-old patient complains of pain and bleeding during defecation. Digital rectal
investigation revealed a tumour of the anal canal. After verification of the diagnosis the patient
was diagnosed with squamous cell carcinoma. The secondary (metastatic) tumour will be most
probably found in:

100% Lungs

0% Liver

0% Pelvic bones

0% Mediastinum
0% Brain

Survey radiograph of chest of a 62 year old smoker who often suffers from "pneumonias"showed a
triangle shadow in the right lung, its vertex is pointing towards the lung root. It also showed devi-
ation of heart and mediastinum shadows towards the lesion. What is the most probable
diagnosis?

100% Cenral cancer of lung

0% Lung abscess

0% Lung cyst

0% Peripheral cancer of lung

0% Atelectasis

INFLUENZA
To replace the placeholder text on this page, you can just select it all and then start typing. But
don’t do that just yet!

MCQS

A 40-year-old patient suffers from influenza. On the 5th day of illness there are pain behind
sternum, cough with sputum, inertness. Temperature is 39,5°C. Face is pale. Mucosa of
conjunctivas and pharynx is hyperemic. Heart rate is 120/min, breathing rate is 38/min. In the
lower lung segments shortening of percussion sound and moist rales (crackles) can be detected.
What additional investigation should be performed first of all to specify the diagnosis?

100% Lung X-ray

0% ECG

0% Heart US

0% Mantoux test

0% Spirography
During winter epidemics of influenza caused predominantly by virus A/California/04/2009
(H1N1),

on the 2nd day after the disease onset a 30-year-old hospitalized man presented with high fever,

dry cough, myalgia, headache, and general weakness. What should be prescribed as an etiotropic

treatment in this case?

Neuraminidase inhibitors (oseltamivir)*

Immunoglobulin

Interferon inducers

Antibiotics

Acyclovir

An ambulance team has brought a man diagnosed with acute respiratory viral infection into the
inpatient department. The onset of the disease was acute, with fever of 39.9oC. The patient is
complaining of a headache in his fronto-temporal regions, pain in the eyeballs, aches all over his
body, stuffed nose, sore throat, and dry cough. At home, he had two episodes of nosebleeds. What
type of acute respiratory viral infection does this patient have?

Influenza*

Parainfluenza

Enterovirus infection

Adenovirus infection

Respiratory syncytial infection

Ambulace brought to the hospital a patient with acute respiratory viral infection. The illness
began suddenly with temperature rise up to 39,90C. He complains of headache in frontotemporal
lobes, pain in eyeballs, aching of the whole body, nose stuffiness, sore throat, dry cough. At home
he had a nasal hemorrhage twice. What type of acute respiratory viral infection is it?

100% Influenza

0% Adenoviral infection

0% Parainfluenza
0% RS-infection

0% Enterovirus infection

PERTUSIS
To replace the placeholder text on this page, you can just select it all and then start typing. But
don’t do that just yet!

MCQS

The 5-year-old child has been ill for 2 weeks. Cough attacks developed first and were then

followed by reprises. During coughing the child’s face turns red and cervical veins bulge. The
cough

attacks induce vomiting. X-ray shows intensified bronchial pattern. Blood test: leukocytes-16 x

109/L, lymphocytes-72%, erythrocyte sedimentation rate-4mm/hour. What is the most likely

diagnosis?

Obstructive bronchitis

Foreign body

Pneumonia

Pertussis*

Adenovirus infection

An infant is 2,5 months old. The onset of the disease was gradual, the child had normal body
temperature but presented with slight cough. Within a week the cough intensified, especially at
night; on the 12th day the child developed cough fits occurring up to 20 times per day and
followed by vomiting. There was one instance of respiratory arrest. Make the diagnosis:
100% Pertussis

0% Parainfluenza

0% Congenital stridor

0% Respiratory syncytial infection

0% Adenovirus infection

RESPIRATORY FAILURE
To replace the placeholder text on this page, you can just select it all and then start typing. But
don’t do that just yet!

MCQS

A 54 y.o. male patient suffers from dyspnea during mild physical exertion, cough with sputum
which is excreted with diffculty. On examination: diffuse cyanosis. Is Barrel-chest. Weakened
vesicular breathing with prolonged expiration and dry whistling rales. Ap is 140/80 mm Hg, pulse
is 92 bpm, rhythmic. Spi-rography: vital capacity (vC)/predicted vital capacity- 65%, FEV1/FVC-
50%. Determine the type of respiratory insufficiency (RI).

100% RI of mixed type with prevailing obstruction

0% RI of restrictive type

0% RI of obstructive type

0% RI of mixed type with prevailing resri-ction

0% There is no RI

A 19 y.o. girl was admitted to Emergency Department: unconsionsness state, cyanosis, myotic
pupil, superficial breathing - 12/min. BP- 90/60 mm Hg, Ps-78/min. Choose the action
corresponding to this clinical situation:

100% Controlled respiration

0% Gastric lavage
0% Oxygen inhalation

0% Caffeine injection

0% Cordiamine injection

A 7-year-old patient presents with body temperature rise up to 39oC, dry cough, pain in the lateral
abdomen. Objectively: there is cyanosis of the nasolabial triangle, inspiratory dyspnea with
accessory muscle recruitment. Percussion reveals pulmonary dullness; among auscultation
findings there are diminished breath sounds, crepitant rales. Respiratory rate is of 50/min, HR-
120/min. Evaluate the grade of respiratory failure in the patient:

100% II

0% I

0% III

0% IV

0% 0

ALVEOLITIS
To replace the placeholder text on this page, you can just select it all and then start typing. But
don’t do that just yet!

MCQS

A 52-year-old patient suffers from marked dyspnea during physical exertion, non-productive
cough. The patient’s condition has been persisting for 8 months. The patient has been a smoker
for 30 years. In the lungs there are cellophane-type crackles auscultated on both sides. Respiration
rate is 26/min., oxygen saturation of blood is 92%. On spirometry: moderate restrictive-type
disturbance of external respiration. What is the most likely diagnosis?

100% Idiopathic fibrosing alveolitis


0% Chronic obstructive pulmonary disease (COPD)

0% Chronic bronchitis

0% Community-acquired pneumonia

0% Sarcoidosis

Physiology of pregnancy
To replace the placeholder text on this page, you can just select it all and then start typing. But
don’t do that just yet!

MCQS

A famil

Physiology of pregnancy
To replace the placeholder text on this page, you can just select it all and then start typing. But
don’t do that just yet!

MCQS

A famil
Physiology of pregnancy
To replace the placeholder text on this page, you can just select it all and then start typing. But
don’t do that just yet!

MCQS

A famil

Physiology of pregnancy
To replace the placeholder text on this page, you can just select it all and then start typing. But
don’t do that just yet!

MCQS

A famil

Physiology of pregnancy
To replace the placeholder text on this page, you can just select it all and then start typing. But
don’t do that just yet!

MCQS

A famil
Physiology of pregnancy
To replace the placeholder text on this page, you can just select it all and then start typing. But
don’t do that just yet!

MCQS

A famil

SHOCK
To replace the placeholder text on this page, you can just select it all and then start typing. But
don’t do that just yet!

MCQS

After a 10-year-old child had been bitten by a bee, he was delivered to a hospital. There were lip,
face and neck edemata. The patient felt hot and short of breath. Objectively: breathing was
laboured and noisy. There were foamy discharges from the mouth, cough. The skin was pale and
cold. There was bradypnoea. Heart sounds were muffled and arrhythmic. Thready pulse was
present. What diagnosis was made by the expert in resuscitation?

100% Anaphylactic shock

0% Quincke’s edema

0% Bronchial asthma

0% Acute cardiovascular collapse

0% Cerebral coma
HEART FAILURE
To replace the placeholder text on this page, you can just select it all and then start typing. But
don’t do that just yet!

MCQS

At night a 63-year-old woman suddenly developed an asphyxia attack. She has a

15year-long history of essential hypertension and had a myocardial infarction 2 years

ago. Objectively her position in bed is orthopneic, the skin is pale, the patient is

covered with cold sweat, acrocyanosis is observed. Pulse – 104/min. Blood pressure –

210/130 mm Hg, respiration rate – 38/min. Pulmonary percussion sound is clear, with

slight dullness in the lower segments, throughout the lungs single dry crackles can be

heard that become bubbling and non-resonant in the lower segments. What is the most

likely complication in this patient?

Acute left ventricular failure*

Acute right ventricular failure

Pulmonary embolism

Bronchial asthma attack

Paroxysmal tachycardia

After myocardial infarction, a 50-year-old patient had an attack of asthma. Objectively: bubbling
breathing with frequency of 32/min, cough with a lot of pink frothy sputum, acrocyanosis,
swelling of the neck veins. Ps- 108/min, AP- 150/100 mm Hg. Heart sounds are muffled. Mixed
moist rales can be auscultated above the entire lung surface. What drug would be most effective in
this situation?

100% Nitroglycerin intravenously

0% Pentamin intravenously
0% Strophanthin intravenously

0% Dopamine intravenously

0% Aminophylline intravenously

CARDIOGENIC SHOCK
To replace the placeholder text on this page, you can just select it all and then start typing. But
don’t do that just yet!

MCQS

A 64 y.o. patient has developed of squeering substernal pain which had appeared 2 hours ago and
irradiated to the left shoulder, marked weakness. On examination: pale skin, cold sweat. Pulse- 108
bpm, AP- 70/50 mm Hg, heart sound are deaf, vesicular breathing, soft abdomen, painless,
varicouse vein on the left shin, ECG: synus rhythm, heart rate is 100 bmp, ST-segment is sharply
elevated in II, III aVF leads. What is the most likely disorder?

100% Cardiogenic shock

0% Cardiac asthma

0% Pulmonary artery thromboembolia

0% Disquamative aortic aneurizm

0% Cardiac tamponade

Complicated hypertensic crisis


To replace the placeholder text on this page, you can just select it all and then start typing. But
don’t do that just yet!
MCQS

A 58-year-old patient complains of a headache in the occipital region, nausea, choking,


opplotentes. The presentations appeared after a physical exertion. Objectively: the patient is
excited. Face is hyperemic. Skin is pale. Heart sounds are regular, the 2nd aortic sound is
accentuated. BP- 240/120 mm Hg, HR- 92/min. Auscultation reveals some fine moist crackles in
the lower lungs. Liver is not enlarged. ECG shows signs of hypertrophy and left ventricular
overload. What is the most likely diagnosis?

100% Complicated hypertensic crisis

0% Acute myocardial infarction, pulmonary edema

0% Bronchial asthma exacerbation

0% Uncomplicated hypertensic crisis

0% Community-acquired pneumonia

You might also like